Specialties - O&G Flashcards

A few of these questions are taken from '450 SBAs in Clinical Specialities' which is an amazing resource. Those questions have an acknowledgement in the answer

1
Q

Which of the following describes the first test typically used to screen pregnant women, that could indicate the risk of Down’s syndrome?

A. An USS performed at 20 weeks to assess for 11 major rare disorders
B. An USS performed between 11-14 weeks which assesses for associated cardiac and facial abnormalities
C. A sample of amniotic fluid taken from the 15th week which is analysed to detect fetal genetic or chromosomal abnormalities
D. Serum B-hCG and PAAP-A supplemented by an USS
E. A sample of placental villus taken from the 11th week which is analysed to detect fetal genetic or chromosomal abnormalities

A

D. Serum B-hCG and PAAP-A supplemented by an USS

As standard, the screening program for pregnant women is as follows:

11-14 weeks - dating scan and combined Down’s syndrome test. The dating scan itself can be done between 8-14 weeks, but this is restricted to 11-14 weeks if the mother wishes to screen for Down’s syndrome.

20 week - a USS to look for 11 rare but serious fetal abnormalities.

How well did you know this?
1
Not at all
2
3
4
5
Perfectly
2
Q

A 13 year old girl presents to a GUM clinic with concerns she may be pregnant. She reveals she has recently become sexually active with an older boy at her school. A pregnancy test is negative. She also says she has noticed some suspicious discharge.

What is the most important step for the doctor to take?

A. Counsel her on safe sexual practice in the future, and inform a colleague about your decision to maintain her privacy
B. Ask further questions about the nature of the relationship to ascertain whether there is any abuse
C. Advise her that she should use contraception, but it cannot be prescribed unless she returns accompanied by a parent or guardian
D. Inform the police, as is a legal imperative in a case of sexual activity in a child
E. Assess the girl’s capacity to consent, with a view to prescribing antibiotics if she is competent

A

B. Ask further questions about the nature of the relationship to ascertain whether there is any abuse

A doctor has no legal obligation to report underage sexual activity UNLESS there are signs of abusive or harmful activity. ‘A’ is a good answer in that she should be counselled, and the decision to maintain privacy of a young child in this case should be shared with a designated doctor, but the relationship should be assessed for harm or abuse before making the decision to preserve confidentiality.

‘C’ is untrue, as providing the child satisfies the requirements of the Fraser guidelines, they can be given contraception. ‘D’ is untrue because you are not necessarily legally compelled to break confidentiality here; you need to assess the nature of the relationship, and if it is abusive or harmful you would be compelled to break confidentiality.

‘E’ is true, as the child may be found to be Gillick competent, but not the biggest concern at the time.

NB: the difference between Gillick competence and Fraser guidelines

How well did you know this?
1
Not at all
2
3
4
5
Perfectly
3
Q

A 27 week pregnant woman presents to the maternal assessment unit with bright red vaginal spotting. She says this has happened twice over the past 3 weeks. She denies any pain but is extremely concerned for her baby.

What is the most appropriate next step in her management?

A. Admit to the antenatal ward, establish IV access, consider steroids, and employ a watch and wait approach
B. Perform an USS to determine the location of the placenta
C. Admit to the antenatal ward, attach a CTG probe, and take FBC and G&S
D. Take a focused history and basic obs, and perform a digital vaginal examination
E. Admit to the labour ward, inform a senior obstetrician, and take FBC, LFTs, U&Es, Coagulation screen, and X-match for 4 units

A

C. Admit to the antenatal ward, attach a CTG probe, and take FBC and G&S

This is a history of placenta praevia and, though an USS will confirm the diagnosis, it is more urgent to first establish the welfare of the fetus and take bloods to help predict and address haemorrhage. FBC, and G&S should always be taken, along with clotting studies and cross match depending on the severity of the bleeding and the clinical background.

A history of a painless bleed on a background of previous smaller bleeds is suggestive of placenta praevia - where the placenta lies in the lower segment of the uterus and may obstruct the internal cervical os. Placenta praevia complicates only 0.5% of pregnancies at term, but 10 times as many seem to feature a low-lying placenta at 20 weeks. This is because after 20 weeks the lower segment of the uterus expands, hence 90% of low-lying placentas then appear to move up.

The most obvious complication of placenta praevia is that it will obstruct the descent of the baby into the pelvis, and so will impair vaginal delivery.

Once the immediate health of mother and baby has been established, the source of the bleeding is identified, and the bleeding has stopped, the mother can potentially be sent home.

VAGINAL EXAMINATION SHOULD NEVER BE CARRIED OUT IN A PREGNANT WOMAN BLEEDING VAGINALLY WITHOUT EXCLUDING PLACENTA PRAEVIA - IT CAN PROVOKE MASSIVE BLEEDING.

How well did you know this?
1
Not at all
2
3
4
5
Perfectly
4
Q

Which of the following CTG traces would be considered pathological?

A. Decelerations with varying recovery periods in a woman who is 10cm dilated and has begun to push
B. Decelerations of 35bpm that occur only with contractions
C. Fetal heart rate of 105bpm in a 42+3 week baby with a transverse lie
D. Spikes in fetal heart rate from 150bpm to 170bpm occurring with uterine contractions
E. A stable fetal heart rate of 130bpm that has not varied by more than 5bpm for the last hour

A

E. A stable fetal heart rate of 130bpm that has not varied by more than 5bpm for the last hour

According to NICE guidelines, CTG traces can be classified as: normal, suspicious, pathological, or showing a need for urgent intervention. These categories are defined by the number of reassuring, non-reassuring, and abnormal CTG characteristics that appear:

Normal: all features are reassuring
Suspicious: 1 non-reassuring feature
Pathological: 1 abnormal feature OR 2 non-reassuring features
Needs urgent intervention: acute bradycardia, or a single deceleration lasting 3 minutes or more

How well did you know this?
1
Not at all
2
3
4
5
Perfectly
5
Q

A g3p0 pregnant woman is found to be Rh- at her 10 week booking appointment. The subsequent test for Rh D antibodies is positive, thought to be due to sensitisation in her 2 previous miscarried pregnancies. Her D-antibody levels are measured every 4 weeks, and she is referred to a fetal medicine specialist who monitors for fetal anaemia.

Which is the most appropriate method to monitor for fetal anaemia?

A. Regular USS to monitor the volume of amniotic fluid
B. Umbilical vein blood sampling
C. Fortnightly blood tests for fetal bilirubin
D. Continuous fetal heart monitoring past the 20th week of pregnancy
E. Doppler ultrasound scan of the middle cerebral artery

A

E. Doppler ultrasound scan of the middle cerebral artery

An MCA doppler is useful to monitor for fetal anaemia because of the physiological response to anaemia: the fetus will protect its most vital organs (particularly the brain) and so will increase its cardiac output, and especially the flow to the brain. This can be detected on a doppler scan of the middle cerebral artery (MCA). If fetal anaemia is suspected, umbilical vein sampling can be used to quantify the anaemia, though this carries a 1% chance of fetal loss.

How well did you know this?
1
Not at all
2
3
4
5
Perfectly
6
Q

Which of the following locations is endometriosis least likely to affect?

A. Fallopian tubes
B. Vesico-uterine pouch
C. Utero-sacral ligaments
D. Rectum
E. Recto-uterine pouch
A

D. Rectum

Endometriosis is defined as growth of endometrial tissue outside of the uterus. The most commonly affected sites are the recto-uterine pouch (Pouch of Douglas), the vesico-uterine pouch, the utero-sacral ligaments, the ovaries, and the fallopian tubes. Endometriosis can spread to affect any pelvic or abdominal organ, though this is less likely.

Endometriosis is an historically under-diagnosed and underestimated disease: it is diagnosed in 1-2% of women, but is estimated to affect 10%. Endometriomas are functionally similar to endometrium, meaning they become painful and bleed once a month. This cycle of inflammation and bleeding leads to painful irritation of the peritoneum and scarring. If this scarring occurs on the fallopian tubes or ovaries, it can impair fertility, and indeed the prevalence of endometriosis in subfertile women is 30-50%.

Even more importantly, endometriosis causes chronic pelvic pain. The pain usually begins ~4 days before the woman’s period, and is poorly localised in the pelvis or abdomen. The pain is not accompanied by excess vaginal bleeding as the affected locations do not communicate with the vagina, though PR bleeding may occur if the rectum or colon are involved. The impact of the chronic pain of endometriosis on quality of life is massive, so it is worth keeping this differential in mind for pain presentations.

Treatment is with hormonal options (Mirena coil, COCP, progesterone preparations, temporary menopause with GnRH agonist), analgesia, and laparoscopic removal of lesions if necessary.

How well did you know this?
1
Not at all
2
3
4
5
Perfectly
7
Q

Which of these diseases is likely to reduce in severity during pregnancy?

A. Rheumatoid arthritis
B. Asthma
C. Systemic lupus erythematosus
D. Scleroderma
E. Atopic dermatitis
A

A. Rheumatoid arthritis

During pregnancy there is a shift in the body’s immune system away from Th1-mediated responses, towards Th2 responses. This means that Th1-mediated diseases such as rheumatoid arthritis and multiple sclerosis will improve during pregnancy (though they will tend to rebound sharply afterwards).

How well did you know this?
1
Not at all
2
3
4
5
Perfectly
8
Q

A woman with a diagnosis of PCOS attends her local GP surgery asking for advice on getting pregnant, and how her condition will affect that.

Which of the following statements is false?

A. In PCOS women with insulin resistance, metformin improves live healthy birth rate
B. She is more likely to experience first trimester miscarriages than unaffected women
C. She is at no increased risk of pre-eclampsia
D. Letrozole is an alternative to clomiphene for ovulation induction with a lower chance of causing multiple pregnancy
E. Weight loss and diet control alone improve pregnancy outcomes

A

C. She is at no increased risk of pre-eclampsia

How well did you know this?
1
Not at all
2
3
4
5
Perfectly
9
Q

Which of the following CTG traces would be considered suspicious (by NICE guidelines)?

A. Early decelerations occurring with contractions, with a fetal heart rate of 150bpm
B. Rapid decelerations with varying recovery periods present for half an hour, with a heart rate of 130bpm and baseline variability of 15
C. Decelerations that start at the peak of uterine contraction and recover after the contraction ends, persisting for 40 minutes
D. A smooth, regular, wave-like pattern on the CTG cycling at a frequency of ~4Hz
E. Baseline heart rate of 105bpm with a baseline variability of 10bpm

A

E. Baseline heart rate of 105bpm with a baseline variability of 10bpm

A baseline heart rate of 100-110bpm is a non-reassuring trait, but the baseline variability is within the acceptable range of 5-25 bpm (all based on NICE guidelines). The presence of 1 non-reassuring trait makes this a suspicious CTG.

A and B are reassuring
C and D are pathological

How well did you know this?
1
Not at all
2
3
4
5
Perfectly
10
Q

A nulliparous 32 year old woman visits a fertility specialist regarding her trouble in conceiving. She has a PMHx of chronic pelvic pain, and has previously been investigated for possible pelvic inflammatory disease, IBS, and cystitis to no avail. She states that her pain is not constant, but begins four days before her period and lasts for the period’s duration. She also experiences deep dysparenunia, which ultimately led to her last break up and an inability to form any new relationships. On vaginal exam there are no abnormalities, though the patient reports deep tenderness. Her AMH is normal, and USS of the ovaries shows a good number of antral follicles, though hysterosalpingography shows impaired filling of the Fallopian tubes.

Given the likely diagnosis and the patient’s history, which of the following is the most appropriate management?

A. Hysterectomy with bilateral salpingo-oopherectomy
B. Laparoscopic destruction of endometrial lesions
C. NSAIDs and paracetamol, with potential for addition of opiates
D. Continuous COCP use
E. Implantation of the Mirena coil

A

B. Laparoscopic destruction of endometrial lesions

This case requires a management that alleviates the chronic pain but preserves fertility and allows this woman to continue trying to get pregnant, hence ‘A’, ‘D’ and ‘E’ are unsuitable. ‘C’ seems a fairly inadequate response for severe chronic pain, which this woman has probably tried using OTC analgesia for. Opiates are a bad idea for use in chronic pain.

Endometriosis is defined as growth of endometrial tissue outside of the uterus. The most commonly affected sites are the recto-uterine pouch (Pouch of Douglas), the vesico-uterine pouch, the utero-sacral ligaments, the ovaries, and the fallopian tubes. Endometriosis can spread to affect any pelvic or abdominal organ, though this is less likely.

Endometriosis is an historically under-diagnosed and underestimated disease: it is diagnosed in 1-2% of women, but is estimated to affect 10%. Endometriomas are functionally similar to endometrium, meaning they become painful and bleed once a month. This cycle of inflammation and bleeding leads to painful irritation of the peritoneum and scarring. If this scarring occurs on the fallopian tubes or ovaries, it can impair fertility, and indeed the prevalence of endometriosis in subfertile women is 30-50%.

Even more importantly, endometriosis causes chronic pelvic pain. The pain usually begins ~4 days before the woman’s period, and is poorly localised in the pelvis or abdomen. The pain is not accompanied by excess vaginal bleeding as the affected locations do not communicate with the vagina, though PR bleeding may occur if the rectum or colon are involved. The impact of the chronic pain of endometriosis on quality of life is massive, so it is worth keeping this differential in mind for pain presentations.

Treatment is with hormonal options (Mirena coil, COCP, progesterone preparations, temporary menopause with GnRH agonist), analgesia, and laparoscopic removal of lesions if necessary.

How well did you know this?
1
Not at all
2
3
4
5
Perfectly
11
Q

Which of the following is the most important diagnostic tool for ectopic pregnancy?

A. Trans-vaginal USS
B. Trans-abdominal USS
C. One off serum beta-hCG
D. Serial serum beta-hCGs
E. MRI
A

A. Trans-vaginal USS

Serial beta-hCGs may be useful in guiding treatment, but TVUSS is the diagnostic investigation of choice. One-off beta-hCG should be used to confirm the diagnosis alongside TVUSS, and MRI may be useful in diagnosing Caesarean scar pregnancies.

How well did you know this?
1
Not at all
2
3
4
5
Perfectly
12
Q

Define placenta accreta, increta, and percreta

A

Accreta - the placenta invades past the basement membrane of the decidua (endometrium) and attaches to the myometrium
Increta - the placenta invades into the myometrium
Percreta - the placenta invades through the full thickness of the myometrium and may invade local organs

How well did you know this?
1
Not at all
2
3
4
5
Perfectly
13
Q

Cord prolapse occurs after rupture of membrane in 0.2% of births when the cord descends and becomes compressed. It is an emergency and can lead to fetal hypoxia if not dealt with.

Which of the following most increases the risk of cord prolapse?

A. Advanced maternal age
B. Fully extended breech position
C. Oligohydramnios
D. Preeclampsia
E. A transverse lie
A

E. A transverse lie

Cord prolapse is particularly associated with a transverse lie because that position creates the space through which the cord may exit the uterus. Though breech position increases the risk, at least in a fully extended breech the baby mostly obscures the internal cervical os.

Cord prolapse is an obstetric emergency as it may lead to rapid fetal hypoxia and death. The mother should be placed either with her knees to her chest, or in the Trendelenburg position (supine and with a feet elevated incline) and the presenting part of the baby should be elevated to avoid cord compression. Urgent delivery is necessary, generally via Caesarean section.

How well did you know this?
1
Not at all
2
3
4
5
Perfectly
14
Q

Which of the following statements comparing chorionic villus sampling (CVS) with amniocentesis is correct?

A. Amniocentesis is not necessarily accurate due to a risk of placental mosaicism of fetal cells
B. CVS carries ~3% risk of miscarriage, whereas amniocentesis has ~1% risk
C. Amniocentesis features as part of routine screening for pregnancies, but CVS does not
D. CVS can be carried out 4 weeks earlier than amniocentesis
E. Amniocentesis has the advantage of being performed earlier, giving the mother more time to make decisions

A

D. CVS can be carried out 4 weeks earlier than amniocentesis

CVS and amniocentesis are two tests used to look for genetic fetal abnormalities, and both carry a 1% risk of causing spontaneous miscarriage of a pregnancy. Neither one is considered superior in terms of accuracy of diagnosis, though there is a risk with CVS that mosaic cells (genetically different lineage) in the placenta will be sampled giving a false diagnosis.

CVS is usually performed between 11-14 weeks, and amniocentesis between 15-20 weeks; both can be performed later if needed, but not earlier. because of a higher risk of pregnancy loss. Neither one is part of a routine screening program, but is offered to pregnant women if there is considered to be a high risk of a genetic condition. CVS is arguably superior because it can be done earlier and so gives parents more time to make decisions about the pregnancy.

How well did you know this?
1
Not at all
2
3
4
5
Perfectly
15
Q

Which bloods and measurements should be taken as standard in someone with suspected preeclampsia

A
Blood pressure
24 hour urinary protein or dipstick or PCR
Serum urea, creatinine, and uric acid
FBC
LFTs
U&Es
PTT
How well did you know this?
1
Not at all
2
3
4
5
Perfectly
16
Q

What is the underlying pathology of polyhydramnios?

A

Either increased fetal urine production (gestational diabetes, TTTS) or reduced fetal swallowing (duodenal atresia, bowel malformation, chromosomal abnormality, neurological issue)

Infection also associated

How well did you know this?
1
Not at all
2
3
4
5
Perfectly
17
Q

You examine the abdomen of a 32 week pregnant woman.

Which of the following findings would be abnormal?

A. A hyperpigmented line running down the midline of the abdomen
B. Being able to palpate five 5ths of the fetal head
C. A fundal height of 28cm
D. A fetal heart rate of 160bpm
E. A cephalic presentation

A

C. A fundal height of 28cm

The general rule for fundal-symphysial height is that from 24 weeks, it is equal to the gestation +2cm.

How well did you know this?
1
Not at all
2
3
4
5
Perfectly
18
Q

What is the underlying pathology of oligohydramnios?

A

Reduced amniotic fluid because of either loss (rupture of membranes) or decreased production (TTTS, urinary tract pathology, placental insufficiency)

How well did you know this?
1
Not at all
2
3
4
5
Perfectly
19
Q

Describe the pathophysiology of preeclampsia

A

Insufficient spiral artery remodelling leads to a lack of placental blood flow. This causes placental hypoxia, causing syncytiotrophoblast microparticles to be shed into the maternal circulation which causes systemic endothelial damage. This reduces NO and prostacyclin production leading to vasoconstriction, and also makes vessels leaky, leading to oedema and proteinuria.

How well did you know this?
1
Not at all
2
3
4
5
Perfectly
20
Q

A 17 year old girl presents to A&E with lower abdominal pain. She reports her periods have not been regularly coming, though she has had some irregular bleeding.

What is the most appropriate next step?

A. Take LFTs and U&Es
B. USS of the pouch of Douglas
C. TVUSS of the ovaries
D. Take a clotting screen, G&S, and X-match
E. Take a urinary b-HcG
A

E. Take a urinary b-HcG

Whilst there are a range of possible differentials for this presentation, a very important one is ectopic pregnancy, and this can be tested for very quickly and easily using a urinary pregnancy test.

How well did you know this?
1
Not at all
2
3
4
5
Perfectly
21
Q

A 29-year-old woman is seen at her booking visit and has blood taken for screening.

Which of these is the most appropriate set of booking tests?

A. Hepatitis C, human immunodeficiency virus (HIV), syphilis and toxoplasmosis
B. Rubella, hepatitis B, hepatitis C and syphilis
C. Syphilis, hepatitis B and HIV
D. HIV, cytomegalovirus, rubella and hepatitis B
E. HIV, syphilis, rubella and group B Streptococcus

A

C. Syphilis, hepatitis B and HIV

The serum tests for infection that NICE recommend as an offer at booking are syphilis, HIV, and hepatitis B (C). Cytomegalovirus (D) is a DNA virus that usually leads to asymptomatic infection. Transmission to the fetus leading to damage occurs in about 10 per cent of cases.

Forty to 50 per cent of all women of childbearing age have not had cytomegalovirus infection so it is not cost effective to screen everyone. Toxoplasmosis is contracted from such things as undercooked/cured meat and cat faeces. It is not routinely tested for in pregnancy as the low risk of toxoplasmosis (A) becoming a florid infection rather than an indolent disease in a non-immunocompromised infection makes it not worthwhile. It is not cost effective to test for hepatitis C (B).

Rubella used to be screened for as maternal infection in the first trimester causes 20% pregnancy loss and 90% congenital syndrome in the surviving babies. however Rubella is now rare enough that it is not cost effective to screen for it.

NB: This question is modified from ‘450 SBAs in Clinical Pathology’

How well did you know this?
1
Not at all
2
3
4
5
Perfectly
22
Q

Which of the following is not a physiological change in pregnancy?

A. 20-30% increase in red cell mass but up to 50% increase in plasma volume by term causing a dilutional anaemia
B. Increase tidal volume and minute ventilation due to the effects of progesterone
C. A drop in cardiac output immediately after delivery
D. A reduction in peripheral vascular resistance
E. 40% increase in cardiac output by 20 weeks

A

C. A drop in cardiac output immediately after delivery

Immediately after delivery CO increases to 60-80% more than pre-pregnancy because of the alleviation of pressure on the IVC by the gravid uterus. CO then rapidly returns to pre-pregnancy values within an hour or so post-partum.

Arterial pressure may decrease in mid-pregnancy, but will recover in the third trimester.

How well did you know this?
1
Not at all
2
3
4
5
Perfectly
23
Q

Which of the following options describes the start of labour?

A. From when the cervix reaches 10cm dilated
B. From the beginning of cervical dilatation accompanied by painful contractions
C. From when contractions reach a frequency of 3 in 10 minutes
D. From when the head is fully engaged in the pelvis (two fifths or less of the head palpable)
E. Rupture of the membranes

A

B. From the beginning of cervical dilatation accompanied by painful contractions

Labour is considered to start when painful contractions and cervical dilatation begin. This marks the start of the latent phase of the first stage of labour, which is generally slow. Once cervical dilatation reaches 4cm, the cervix dilates more rapidly, by about 1cm/hr in nulliparous women, and 2cm/hr in multiparous women. This second part is known as the active phase, and once a woman enters this phase they are said to be ‘in established labour’. The first stage of labour is complete when the cervix dilates to 10cm.

The second stage then covers the time from full dilation to delivery of the baby. The passive stage comes first, where the baby’s head descends into the pelvic floor. Once the mother feels the urge to push, the active stage begins. In practice the management of this stage will differ, but often women will be given ‘a passive hour’ from the time they are fully dilated to allow the baby to descend into the pelvis. Once the active stage begins, the baby is generally delivered relatively quickly (average 40 minuts in nullips, 20 minutes in multips) though it varies. If the active stage lasts over an hour, spontaneous vaginal delivery becomes unlikely and other options should be considered. However this stage may also resolve extremely quickly (the fastest I have seen is an active stage of 6 minutes).

The final stage is from delivery of the baby to delivery of the placenta. Up to 500mL of blood loss is considered acceptable in a vaginal birth. It is important to check that the whole placenta has been delivered, as it is surprisingly rigid and may hold open sections of the uterus. This is concerning because the uterus needs to clamp down after parturition in order to clamp off the spiral arteries, as they have been remodelled so that they cannot constrict. The uterus should be felt after parturition to feel whether it has contracted: if it has not, this is called uterine atony and they are at risk of post-partum haemorrhage.

How well did you know this?
1
Not at all
2
3
4
5
Perfectly
24
Q

A 32-year-old HIV positive woman who booked for antenatal care at 28 weeks gestation arrives on the delivery suite at 37 weeks with painful regular contractions and a cervix dilated to 4 cm. Ultrasonography confirms a breech singleton pregnancy with a reactive fetal heart rate.

What is the most appropriate management option?

A. Await onset of labour, avoid operative delivery, wash the baby at delivery
B. Induce labour with synthetic prostaglandins
C. Await onset of labour, but have a low threshold for expediting vaginal delivery using forceps
D. Await onset of labour, avoid operative delivery, administer steroids to the infant immediately after birth
E. Caesarean delivery, wash the baby at delivery

A

E. Caesarean delivery, wash the baby at delivery

Although knowledge of managing HIV positive pregnant women is beyond the scope of most undergraduate curricula, in this question the presence of HIV infection is largely a distractor. Delivery of HIV positive women aims to lower the risk of vertical transmission and reduce morbidity. Washing the baby shortly after delivery is a part of that strategy. Induction of labour (B) is not indicated unless there is a benefit to expediting delivery, which in the vignette above there is not.

Interventions which increase the risk of maternal/fetal blood transfusion (and therefore vertical transmission), such as amniocentesis, fetal blood sampling or forceps delivery, are avoided in HIV positive women so (C) is incorrect. Giving neonates steroids (D) is not warranted here for any reason. (A) and (E) could both be correct if the woman had a cephalic singleton delivery. However, this woman is at term, not in established labour and has a breech singleton pregnancy. Following publication of the planned vaginal versus caesarean delivery trial in 2000, which demonstrated improved fetal outcomes with caesarean delivery, most centres now exclusively offer elective caesarean section for these mothers. Hence, even if the woman was not HIV pregnancy, (E) would remain the single best answer.

NB: This question is reproduced from ‘450 SBAs in Clinical Pathology’

How well did you know this?
1
Not at all
2
3
4
5
Perfectly
25
Q

A 28 year old woman attends an appointment with her doctor to talk about getting pregnant. She is a known epileptic who has been well-controlled on sodium valproate for the last 5 years (i.e. since diagnosis). She wants to know how pregnancy will affect her condition, and what changes in her management may be needed.

What should the doctor tell her?

A. She will need to take the standard 400mcg dose of folic acid
B. Her medication dose will need to be increased, and then decreased shortly after birth
C. She will probably experience an increase in seizure activity during pregnancy, but this can be mitigated by drug dosage changes and monitoring
D. Sodium valproate given at the lowest therapeutic dose has no association with birth defects
E. She is at no more increased risk of psychiatric issues during pregnancy than a non-epileptic person

A

B. Her medication dose will need to be increased, and then decreased shortly after birth

‘A’ is wrong because women with epilepsy are recommended to take the higher 5mg dose of folic acid, as there is some evidence it reduces the incidence of congenital malformation in children of woman taking anti-epileptic drugs (AEDs).

‘B’ is correct because there is increased renal and hepatic clearance of drugs during pregnancy which, in addition to the haemodilution, necessitates a higher dose of AEDs. However these changes rapidly reverse post-partum, and so any epileptic woman whose AED dose was increased during pregnancy should be reviewed within 10 days post-partum to adjust the dose.

‘C’ is incorrect because most women will not experience seizure deterioration during pregnancy. Overall about 1/3 of epileptic women experience a deterioration during pregnancy, and the best predictor for this is their seizure activity immediately prior to pregnancy - if their epilepsy is well-controlled, they are unlikely to have seizures while pregnant. Epileptic women are particularly vulnerable to seizures during delivery and immediately post-partum, mostly due to the exhaustion and dehydration that accompanies that period.

‘D’ sodium valproate is associated with neural tube defects. There are conflicting opinions as to whether carbamazepine causes neural tube defects depending on who you ask. Lamotrigine is generally considered safe during pregnancy.

‘E’ is incorrect as both certain varieties of epilepsy and certain AEDs carry an increased risk of depression. This means that epileptic mothers are more at risk from depression during and immediately after pregnancy, and should be made aware so they can monitor any symptoms that arise and seek help.

NB: Babies born to mothers taking AEDs particularly need their vitamin K injection if the AED is enzyme inducing

How well did you know this?
1
Not at all
2
3
4
5
Perfectly
26
Q

A 37 week pregnant woman attends labour ward after her waters broke at home. She had been insistent on a home birth, but after several painful hours she has agreed to come to hospital. She is a known epileptic, and has been taking lamotrigine during her pregnancy. Several hours later the midwife checks her, and she is 8cm dilated, but tired and stressed. She begins to have a seizure shortly afterwards.

How should this seizure be managed?

A. Give I.V. Hartmann’s and supplementary oxygen if saturations dip, and take her immediately to theatre for C-section
B. Give tocolytics to reduce impact on the fetus, and wait for the seizure to self-terminate (give I.V. benzodiazepines if still seizing at 15 minutes)
C. Give I.V. lorazepam, consider I.V. phenytoin and tocolytics if seizures and uterine hypertony persist
D. Give rectal diazepam immediately, then repeat at 15 minutes if the seizure fails to terminate
E. Give buccal midazolam and begin an I.V. dextrose infusion

A

C. Give I.V. lorazepam, consider I.V. phenytoin and tocolytics if seizures and uterine hypertony persist

The acute management of a seizure in a pregnant woman is very similar to that of a non-pregnant person having an epileptic seizure.

First line management is to give I.V. lorazepam, which is usually achievable as women in labour should already have at least one cannula in situ. Diazepam can be given I.V. as an alternative. If there is no I.V. access, then rectal diazepam or buccal midazolam can be given, so ‘D’ and ‘E’ are good second line options. If seizures are not controlled by initial steps, I.V. phenytoin or fosphenytoin is given.

I.V. Hartmann’s and supplementary oxygen (A) may well be needed depending on oxygen saturations and hydration status/ blood pressure, but are not the best option here. Tocolytics may be used for a seizure during labour, but only if there is persistent uterine hypertonus due to the seizure and you become concerned for the fetal wellbeing. The CTG should be continuously monitored, and the decision to expedite delivery should be considered.

How well did you know this?
1
Not at all
2
3
4
5
Perfectly
27
Q

A nervous 42-year-old woman presents herself to your antenatal clinic very worried that she has missed the right time to have her combined test for Down’s syndrome screening. She is now 17 weeks pregnant and is very concerned about her age. You counsel her about the appropriate alternative, the quadruple test and arrange to have this done.

Which assays make up the quadruple test?

A. AFP, PAPP-A, inhibin B and beta hCG
B. Unconjugated oestriol, hCG, AFP and inhibin A
C. Beta hCG, PAPP-A, nuchal translucency and inhibin A
D. AFP, inhibin B, beta hCG and oestriol
E. Unconjugated oestriol, PAPP-A, beta hCG and inhibin A

A

B. Unconjugated oestradiol, hCG, AFP and inhibin A

Down’s syndrome screening is offered to all pregnant women in the UK. She is 42 which gives you an age related risk of one in 55 of having a child with Down’s syndrome. Early in the second trimester the combined test is offered. This includes an ultrasound scan of the fetal neck looking at the nuchal translucency (NT) and two blood tests – PAPP-A and beta hCG. This can be reliably performed from 10 to 13 weeks. Ideally, an integrated test using the combined test and the quadruple test can be used to create a Down’s risk.

As she has missed the chance to have an NT, she would only be offered the quadruple test, which is unconjugated oestradiol, total hCG, AFP and inhibin A – Answer (B). The downside of the quadruple test is that it has a 4.4 per cent false-positive rate compared with 2.2 per cent for the combined test and only 1 per cent for the integrated test. In the event of a high risk result, this woman would be offered an amniocentesis to exclude Down’s syndrome and other chromosome abnormalities.

NB: This question is reproduced from ‘450 SBAs in Clinical Pathology’

How well did you know this?
1
Not at all
2
3
4
5
Perfectly
28
Q

A 38 year old woman who is 11 weeks pregnant presents with spotting and lower abdominal cramping pain. Her observations are stable, and a speculum exam shows the cervical os is closed. During the history she states she has not passed any clots or solid products through her vagina. Her observations and history show she is otherwise well, and she has had no other issues or bleeding during this pregnancy. A TVUSS shows a normal CRL for 11 weeks.

What is the best next step?

A. Give a progesterone analogue
B. Prescribe a 10 day course of erythromycin and monitor fetal cardiac activity
C. Perform a serum b-hCG test and use the result to guide management
D. Counsel the woman that this history is indicative of a miscarriage, and discuss options for delivery/removal of products of conception
E. Counsel the woman that the fetus is alive, but there is a 1 in 4 risk of miscarriage

A

E. Counsel the woman that the fetus is alive, but there is a 1 in 4 risk of miscarriage

This is most likely a threatened miscarriage, in which there is bleeding but the os is closed and the fetus is still alive; it carries a 25% of actual miscarriage. Because the fetus is still alive, the USS should show a fetal pole and cardiac activity (detectable on USS from 5-6 weeks onwards).

‘A’ would imply a missed miscarriage - the fetus has died some time prior but the products of conception have not been passed so the mother is unaware. Later in pregnancy, this may be evident as the uterus will be smaller than expected. ‘B’ implies a septic miscarriage, as these are signs of infection, though there is nothing here to suggest this would be the case. ‘C’ is referring to the lower levels of beta-hCG seen in ectopic pregnancy. Ectopic pregnancy is a very important differential to exclude, but occurs in ~1% of pregnancies making miscarriage more likely

Molar pregnancy (D) is a possible differential, but occurs in 0.1% of pregnancies, whereas miscarriage occurs in ~20% of clinical pregnancies, especially in the first trimester, making ‘E’ a more likely finding.

How well did you know this?
1
Not at all
2
3
4
5
Perfectly
29
Q

A 7 week pregnant woman presents to antenatal clinic with nausea and vomiting of 2 weeks duration. She has been vomiting 6 times per day and has had trouble keeping food down at all.

What is the most appropriate next step?

A. Tell her to purchase OTC 1st generation antihistamines and come back in a week
B. Assess her hydration status, weigh her, and take bloods
C. Prescribe her dexamethasone and tell her to buy over the counter oral rehydration salts
D. Establish I.V. access, give Hartmann’s solution, then take a VBG to assess electrolytes
E. Prescribe vitamin supplements and assess the baby’s growth every 2 weeks after the dating scan

A

B. Assess her hydration status, weigh her, and take bloods

Hyperemesis gravidarum can be diagnosed when there is persistent nausea and vomiting in pregnancy (NVP) with the triad of >5% weight loss, electrolyte imbalance, and dehydration.

It usually follows the same pattern as regular morning sickness in that it begins around 4 weeks and usually disappears by 14-20 weeks. UTI, multiple pregnancy, and molar pregnancy can all cause hyperemesis gravidarum, and so should be routinely excluded.

How well did you know this?
1
Not at all
2
3
4
5
Perfectly
30
Q

A g2p1 woman attends the antenatal clinic to discuss options for the birth of her current pregnancy, and for her family planning thereafter. She delivered her previous baby by C-section, which was indicated by a failure to progress in labour, and is very keen for this baby to be delivered naturally.

How should you advise this woman?

A. There is a 1 in 5 chance she will need an emergency C-section anyway
B. A previous C-section using a lower uterine segment incision is a contra-indication to vaginal birth
C. 2 previous C-sections with no vaginal births is an absolute contra-indication to planning a vaginal birth
D. There is a ~2% risk of uterine rupture at the site of the previous C-section scar
E. Her failure to progress in a previous labour, combined with her never having given birth vaginally, make vaginal birth less likely

A

E. Her failure to progress in a previous labour, combined with her never having given birth vaginally, reduces the likelihood of vaginal birth

Vaginal birth after a previous C-section (VBAC) is complex, and each mother must be assessed individually on their suitability for vaginal birth. A successful VBAC carries the lowest risk of all options, but a failed VBAC with emergency C-section carries the highest, therefore the risks need to be appropriately weighed.

Advise her there is a 75% chance of success, but that attempting and failing vaginal birth to then perform an emergency C-section is the riskiest option. There is also an increased (~0.5%) risk of uterine rupture at the site of the C-section scar, which would necessitate a hysterectomy and would probably result in significant haemorrhage.

The best indicator of a successful VBAC is a previous vaginal birth, especially a previous VBAC which makes the odds of another successful VBAC 85-90%.

How well did you know this?
1
Not at all
2
3
4
5
Perfectly
31
Q

How should you monitor for toxicity in a patient receiving magnesium sulphate prophylaxis against eclampsia?

A

Monitor pulse, RR, and blood pressure, and test the patellar tendon reflex
This should be done at least once every 4 hours

How well did you know this?
1
Not at all
2
3
4
5
Perfectly
32
Q

A G1P0 32+0 week pregnant woman presents to A&E after she experienced a sudden gush of clear fluid from her vagina. There was no associated pain or bleeding. On speculum examination the cervix is closed with no bulging membranes and there is a pool of fluid in the posterior fornix. Her obs are recorded, bloods are taken, and a CTG is performed.

SO2: 98% RA
HR: 71bpm
BP: 115/83mmHg
Temp: 36.7
Hb: 108g/L
WCC: 8x10^9/L
CRP: <3mg/L
CTG: fetal heart rate 150bpm, variability 25bpm, some early decelerations

How should this patient be managed?

A. TVUSS to assess the liquor volume, discharge with safety netting if normal
B. Admit her and perform an IGFBP‐1 or PAMG‐1 test
C. Admit her, give erythromycin and steroids, and employ a watchful waiting approach
D. Give steroids and aim to induce labour within 24 hours
E. Continuous CTG monitoring for 24 hours, discharge with safety netting and follow-up if non-concerning

A

C. Admit the woman, give a 10 day course of erythromycin, offer steroids, and employ a watchful waiting approach

In a woman presenting with rupture of the membranes between 24+0 and 36+6 weeks, a speculum exam should be performed. If the speculum exam is not clear (does not show pooled liquor) then an IGFBP‐1 or PAMG‐1 test is a sensitive and specific marker to aid diagnosis. If PPROM is confirmed, the patient should be admitted for at least 48 hours as in >50% of cases preterm labour follows PPROM within 48 hours. Patients with no contraindications should be offered expectant management until 37 weeks with frequent clinical assessment.

Infection often co-exists with PPROM and may be either the result or the cause: the earlier the gestation, the more likely the infection is to have caused PPROM. Infection is an important complication and should be assessed using a combination of maternal history, maternal abdominal exam, CTG monitoring, bloods including a CRP and WCC, and potentially a high vaginal swab (though none of these should be used in isolation). Any woman presenting with PPROM should be given a course of antibiotics for 10 days or until she is in established labour – whichever comes first. Erythromycin is the first-line antibiotic. Co-amoxiclav is contraindicated as it raises the risk of necrotising enterocolitis in the newborn.

A. This patient requires admission and the measurement of amniotic fluid volume in the context of PPROM using ultrasound is not recommended or supported by evidence.

B. This would be a good answer if the speculum examination were less conclusive, but is unnecessary here.

D. There is no reason to induce labour here

E. There is no indication for continuous CTG monitoring and this woman should not be discharged home given the risk she will deliver soon.

NB: “established labour” refers to the second part of the first stage of labour: when the cervix dilates from 4cm to 10cm.

How well did you know this?
1
Not at all
2
3
4
5
Perfectly
33
Q

A 20 week pregnant woman attends antenatal clinic for the 18-21 week abnormality scan. No fetal abnormalities are detected, and baby looks to be growing well, however the placenta is observed to be low-lying, close to the internal cervical os.

What is the most appropriate next step?

A. Counsel the woman that 60% of low-lying placentas do not progress to placenta praevia, and that you will reassess its position later
B. Do nothing for the moment but offer the woman a follow-up scan at 32 weeks
C. Measure the distance from the nearest placental edge to the cervical os; if it is within 20mm, book the woman for Caesarean section at 37 weeks
D. Counsel the mother that vaginal birth will not be safe, and that an elective C-section is the best option
E. Counsel the mother that if the placenta is >10mm from the cervical os at 32 weeks, it is safe to proceed with a vaginal birth

A

B. Do nothing for the moment but offer the woman a follow-up scan at 32 weeks

5% of pregnancies feature a low-lying placenta at 20 weeks, but only 10% of these are still low-lying at term. This is because the lower segment of the uterus expands later than the upper segment, so low-lying placenta usually ‘move up’ after 20 weeks. It is important to determine whether this position will persist, so the mother should be invited for a TVUSS at 32 weeks, and then again at 36 weeks if it is still low-lying. If the placenta is lying directly over the internal cervical os, then it is praevia. If the placenta does not cover the os but is within 20mm, then it is low-lying (this distinction is used because of the difference in risks during delivery). Cervical length should also be assessed at 32 weeks, as this indicates risk of preterm labour and massive haemorrhage during C-section.

She should be counselled that if the placenta is low-lying or praevia, that vaginal birth may be inadvisable or impossible depending on the placenta’s position. She should also be told to attend hospital urgently if she experiences abdominal pain, contractions, or vaginal bleeding. She should be counselled on the risk of massive haemorrhage with delivery, and should be consented to receive blood products and potentially a hysterectomy in the worst case scenario.

How well did you know this?
1
Not at all
2
3
4
5
Perfectly
34
Q

A 33 week pregnant woman presents to A&E with increasingly frequent contractions, and is worried she may be going in to labour. The SHO takes a history, basic obs, and a urinary sample for analysis. They then examine her abdomen, attach a CTG probe (the trace is reassuring), and perform a speculum exam. The cervix is closed, and there is no fluid suggestive of ruptured membranes.

What is the most appropriate next step?

A. Perform a TVUSS to measure cervical length
B. Contact PICU to secure a bed, and arrange a C-section delivery
C. Perform a fetal fibronectin test to assess whether she is in preterm labour
D. Give nifedipine or atosiban along with corticosteroids for fetal tissue maturation
E. Give a bolus of I.V. magnesium sulphate and start an infusion lasting for 24 hours or until birth

A

A. Perform a TVUSS to measure cervical length

In a woman presenting with potential preterm labour, the stage of pregnancy has a small effect on how you diagnose her: if she is less than 30 weeks pregnant and the history and exam suggest preterm labour, then proceed to treatment. If she is 30 weeks pregnant or more, then a TVUSS can be used to assess cervical length: a length of 15mm or less is diagnostic of preterm labour. If TVUSS is contraindicated or unacceptable, then fetal fibronectin can be used instead, but guidelines specifically state not to use the two together. If the cervix were dilated and it was clear she was in labour, this step would be unneccessary.

Once preterm labour is diagnosed, tocolysis can be attempted to stop the labour. First line is nifedipine (CCB) but oxytocin receptor antagonists (e.g. atosiban) can be used if nifedipine is contraindicated. Magnesium sulphate is given I.V. to the mother for neuroprotection of the preterm baby, and corticosteroids should be given to help mature the preterm baby’s lungs.

‘D’ and ‘E’ are therefore both correct treatment options once preterm labour is diagnosed, but are not the best next step. ‘B’ is wrong as there are no indications for C-section, and ‘C’ is unnecessary as there are no contraindications to TVUSS, and there is no need to use the two diagnostic methods together.

How well did you know this?
1
Not at all
2
3
4
5
Perfectly
35
Q

Assuming no prior results were available for comparison, which of the following blood results would be most useful in diagnosing HELLP syndrome?

A. A low haemoglobin
B. Elevated CRP
C. Elevated AST and ALT
D. Low platelets
E. Elevated ALP
A

C. Elevated AST and ALT

Though thrombocytopenia and anaemia are components of HELLP syndrome, they are also both physiological features of pregnancy. In this scenario - assuming no prior results are available for comparison - AST and ALT are more useful because they are a component of HELLP syndrome not physiologically raised in pregnancy. ALP is physiologically raised in pregnancy, and CRP is a very general measure of inflammation.

How well did you know this?
1
Not at all
2
3
4
5
Perfectly
36
Q

Which of the following physiological changes occur in pregnancy?

A. Decreased red cell mass
B. Increased factor XI
C. Decreased MCV
D. Thrombocytopenia
E. Decreased factor VII
A

D. Thrombocytopenia

Thrombocytopenia of the mother (gestational thrombocytopenia) is normal and is thought to occur due to increased consumption rather than a dilutional effect. This is because it manifests late in the pregnancy, unlike the dilutional anaemia.

How well did you know this?
1
Not at all
2
3
4
5
Perfectly
37
Q

Match the following to either oligohydramnios or polyhydramnios depending on which they are associated with.

A. COX inhibitors
B. Fetal renal disease (e.g. Bartter syndrome)
C. Tracheoesophageal fistula
D. Gestational diabetes
E. Anencephaly
F. IUGR
G. Posterior urethral valves
H. Down syndrome
A

Oligohydramnios:
A. COX inhibitors
F. IUGR
G. Posterior urethral valves

Polyhydramnios:
B. Fetal renal disease (e.g. Bartter syndrome)
C. Tracheoesophageal fistula
D. Gestational diabetes
E. Anencephaly
H. Down syndrome
How well did you know this?
1
Not at all
2
3
4
5
Perfectly
38
Q

How does excretion of glucose by the kidneys differ in pregnant women?

A

Glycosuria generally occurs around a serum concentration of 11mmol/L in non-pregnant patients, but will often occur at a lower threshold in pregnancy

How well did you know this?
1
Not at all
2
3
4
5
Perfectly
39
Q

A type I diabetic woman attends her local GP surgery seeking counselling about getting pregnant.

Which of the following statements is false?

A. There is an increased risk of preterm labour and fetal lung immaturity
B. There is a 3-4 times increased risk of neural tube and cardiac abnormality, dependent on peri-conceptual glucose control
C. Diabetics are monitored regularly for preeclampsia, but not routinely given prophylaxis
D. Birth trauma and delivery complications are more likely
E. She may have to take as much as triple her usual medication

A

C. Diabetics are monitored regularly for preeclampsia, but not routinely given prophylaxis

Pre-existing diabetics are at high risk of developing preeclampsia during pregnancy, and so should be given 75-150mg aspirin ODS from 12 weeks till the end of their pregnancy.

Management of diabetics during pregnancy is complex, with myraid potential complications. Established diabetics face more complications than gestational diabetics, with type I and II faring similarly. Maternal complications include: increased birth trauma and delivery complications (due to macrosomia); increased insulin requirement; hypoglycaemia (from attempts to control tightly); UTIs, wound infection, and endometritis; preeclampsia; worsening of pre-existing ischaemic heart disease; and worsening of diabetic retinopathy.

Management of these patients begins pre-conceptually, where the patient is advised to closely monitor their blood glucose levels. Diabetic prospective mothers are advised to take the higher dose of folic acid (5mg instead of 400mcg) for 3 months before conception up until week 12 of pregnancy; this is because of the increased risk of neural tube defects (there is also increased risk of cardiac defects).

Retinal function, renal function, signs of preeclampsia, and fetal growth must all be monitored during pregnancy. Labour is more likely to be complicated, so problems like increased birth trauma and shoulder dystocia should be planned for. Post-partum the baby is particularly vulnerable to hypoglycaemia which must be monitored for. Finally, it is important to note that the mother’s insulin requirement will rapidly return towards pre-pregnancy after delivery, and failure to attenuate insulin doses may lead to hypoglycaemia.

Gestational diabetes in diabetes presenting during pregnancy in susceptible women, due to changes in metabolism. It account for ~90% of diabetes in pregnancy, increase the risk of developing type II diabetes later, and is associated with conditions impairing glucose tolerance e.g. PCOS. An OGTT is performed between 24-28 weeks to assess for gestational diabetes, or earlier (shortly after the booking appointment) if the woman has developed it before.

NB: glycosuria in pregnancy is a less significant finding than normal because a pregnant woman’s kidneys have a lower threshold for excreting glucose (normal being serum concentration of 11mmol/L)

How well did you know this?
1
Not at all
2
3
4
5
Perfectly
40
Q

A 35 year old woman who is 20 weeks pregnant notices a hard painless breast lump and visits her GP. She is referred to specialist breast services, and a carcinoma of the breast is diagnosed. The mother is obviously extremely upset and wants to know more about the disease and how it can be managed.

Which of the following statements regarding breast cancer in pregnancy is true?

A. Systemic chemotherapy is safe from the second trimester onwards
B. The mother needs to make a decision on whether to continue the pregnancy before 24 weeks gestation
C. Pregnancy inherently worsens the prognosis of breast cancer
D. Depending on receptor expression within the tumour, Tamoxifen and Herceptin may be used
E. Radiotherapy is a good and widely used option alongside chemotherapy in breast cancer during pregnancy

A

A. Systemic chemotherapy is safe from the second trimester onwards

‘B’ is not true, for though 24 weeks is the general limit for TOP, it may be carried out at any time if there are risks to the mother’s health.

‘C’ is untrue, though the prognosis of breast cancer is worse in the cohort of pregnant women overall. However this is because pregnant women obviously make up the younger demographic of women with breast cancer, and it is this demographic who are more likely to develop aggressive cancers that do not respond to hormonal treatments.

‘D’ is incorrect as both tamoxifen and Herceptin are contraindicated during pregnancy and breastfeeding.

‘E’ is wrong because radiotherapy is only used if absolutely necessary e.g. to preserve the spinal cord if it is being compressed.

How well did you know this?
1
Not at all
2
3
4
5
Perfectly
41
Q

A 34-year-old woman attends for her booking in her third pregnancy. She had a caesarean section in her first pregnancy 4 years ago and has had a successful vaginal birth after caesarean section (VBAC) 2 years ago. She has a BMI OF 26.

What is the best predictor for a successful VBAC?

A. BMI of less than 30
B. Less than 35 years old
C. Previous vaginal birth
D. Short inter-pregnancy interval
E. Spontaneous onset of labour
A

C. Previous vaginal birth

NB: this is reproduced from a set of practice SBAs for the MRCOG exam

How well did you know this?
1
Not at all
2
3
4
5
Perfectly
42
Q

Which of the following statements about obstetric anal sphincter injury (OASI) during labour is false?

A. Dysuria is a possible complication
B. The woman should wait 3 weeks before attempting any significant activity
C. They occur in 6% of primips, and 2% of multips
D. 60-80% of women with an OASI will experience long-term complications
E. There is an increased risk of recurrence

A

B. The woman should wait 3 weeks before attempting any significant activity

It is recommended to wait 4-6 weeks before attempting significant physical activity after an OASI.

How well did you know this?
1
Not at all
2
3
4
5
Perfectly
43
Q

A 28-year-old woman attends the mental health antenatal clinic at 12 weeks for a booking assessment. This is her first baby.

Which condition gives her the highest risk of puerperal psychosis?

A. Anorexia nervosa
B. Bipolar affective disorder
C. Moderate depression
D. Obsessive compulsive disorder
E. Recurrent anxiety
A

B. Bipolar affective disorder

NB: this is reproduced from a set of practice SBAs for the MRCOG exam

How well did you know this?
1
Not at all
2
3
4
5
Perfectly
44
Q

A 22 year-old-woman presents to the early pregnancy unit with mild left iliac fossa pain. Examination is normal. She has a positive urine pregnancy test. Her serum human chorionic gonadotrophin (hCG) is 700 IU/L. A transvaginal ultrasound scan reports:

‘Bulky anteverted uterus with a 2 mm cystic area centrally located within the endometrial cavity. Both ovaries have normal ultrasonic appearances. There are no adnexal masses or free fluid in the pelvis.’

What is the most appropriate management?

A. Diagnostic laparoscopy +/- proceed
B. Methotrexate therapy
C. Serum hCG (human chorionic gonadotrophin) measurement in 48 hours
D. Serum progesterone
E. Ultrasound scan in seven days
A

C. Serum hCG (human chorionic gonadotrophin) measurement in 48 hours

This is a pregnancy of unknown location (PUL). Ultrasound findings suggest a pseudosac, as a true gestational sac would be eccentrically located and have a double decidual sac sign (two concentric rings surrounding an anechoic sac). The visualisation of the yolk sac is the critical landmark of the gestational sac. Performing serial serum hCG measurements is the next most appropriate step to guide further management.

NB: this is reproduced from a set of practice SBAs for the MRCOG exam

How well did you know this?
1
Not at all
2
3
4
5
Perfectly
45
Q

A 30 week pregnant woman presents to antenatal clinic with intractable itching. It is worst on her palms and soles, and has been badly disturbing her sleep. She has tried taking antihistamines and changing laundry detergent to no avail.

What would be the most appropriate management for this patient?

A. Measure LFTs weekly until delivery, and then again post-partum
B. Prescribe emollients and tell her to take 1st generation antihistamines about 30 minutes before going to bed
C. Advise her this is a reaction in susceptible people to increased levels of oestrogen, but is ultimately harmless
D. Conduct weekly CTG monitoring and USS of the fetus to monitor for complications and chance of stillbirth
E. Monitor bile acids serum concentration at least fortnightly up till delivery, and use the information to guide mode of delivery

A

A. Measure LFTs weekly until delivery, and then again post-partum

‘C’ is correct in that cholestasis of pregnancy is caused by a reaction in susceptible women to increased levels of circulating oestrogen. However it is by no means ultimately harmless, and carries an increased risk of premature birth and meconium passage.

‘B’ would be appropriate for atopic dermatitis, and may provide some relief here, but is not the best option.

‘D’ and ‘E’ are incorrect because CTG, USS, and bile acids serum concentration are not useful in predicting fetal morbidity and mortality, so there is no use in performing them any more regularly than you would for an uncomplicated pregnancy.

NB: there is a common perception that cholestasis of pregnancy increases the risk of stillbirth, but this is not well supported by evidence, and if it exists is thought to be a very slight increased risk

How well did you know this?
1
Not at all
2
3
4
5
Perfectly
46
Q

Which of the following can a standard CTG accurately measure?

A. The intensity of uterine contractions
B. The frequency and duration of uterine contractions
C. The intensity and frequency of uterine contractions
D. The frequency and resting tone of uterine contractions
E. The frequency of uterine contractions

A

E. The frequency of uterine contractions

External transducer CTG traces are considered to only be accurate in assessing the frequency of uterine contractions. The duration and intensity should be assessed by palpating the uterus.

How well did you know this?
1
Not at all
2
3
4
5
Perfectly
47
Q

You are urgently called to assist with a g5p5 woman who has just given birth to twins and is in the middle of post-partum haemorrhage. The midwife estimates blood loss of 1100ml and informs you there is a valid G&S sample and two wide bore cannulas in situ. The patient is semi-conscious with a BP of 85/50 and a HR of 115bpm. The midwife in charge, anaesthetic reg, and obstetric reg have all arrived, and an MOH call has been put out so the transfusion lab and haematologist have been notified.

What is the best next step?

A. Urgently move the woman to theatre and assess surgical options from intrauterine balloon, to artery embolisation, to hysterectomy
B. Immediately give I.V. O- packed RBCs and cryoprecipitate, then begin a syntometrine infusion
C. Rub up the uterine fundus, give I.M. ergometrine, and begin an oxytocin infusion
D. Put her in the head down position, give 15L oxygen, give warmed I.V. Hartmann’s, and take blood for clotting, FBC, U&Es, and cross-match for 6 units
E. Take her to theatre for an examination under anaesthesia to assess tone of the uterus, and plan measures to stem blood loss from there

A

D. Give 15L oxygen, give warmed I.V. Hartmann’s, then take blood for clotting, fbc, U&Es, and cross-match for 6 units

Green Top Guideline on PPH:
https://obgyn.onlinelibrary.wiley.com/doi/full/10.1111/1471-0528.14178

How well did you know this?
1
Not at all
2
3
4
5
Perfectly
48
Q

A 5 week pregnant woman presents with vaginal bleeding. A speculum exam reveals some blood and a closed cervical os. A USS reveals a normally situated placenta and a visible fetal pole. However there is no detectable heartbeat, and the fetal pole is measured at 6mm.

What is the most appropriate next step?

A. Ask her to come back for another scan in a week’s time
B. Advise the woman that based on her LMP dates, a heartbeat should be detectable and this is likely to be a miscarriage
C. Counsel the mother that she has miscarried and discuss medical vs. surgical management
D. Measure the diameter of the gestational sec
E. Reassure the mother that this is absolutely normal at this stage of pregnancy and send her home

A

A. Ask her to come back for another scan in a week’s time

The guidelines for determining whether there is a viable intrauterine pregnancy using TVUSS are as follows:

If there is a fetal heartbeat, there is a viable pregnancy
If there is no heartbeat but there is a fetal pole, measure it
Whether the pole is above or below 7mm, you will likely re-scan in a minimum of 7 days

If there is no pole, measure the gestational sac diameter
Whether the sac diameter is above or below 25mm, you will likely re-scan in a minimum of 7 days

How well did you know this?
1
Not at all
2
3
4
5
Perfectly
49
Q

A 26 week pregnant woman presents with abdominal pain. It is intense, and she came straight to A&E as soon as it started. Abdominal palpation reveals a tense woody feel, and a speculum exam reveals a closed os and no bleeding. Her obs are as follows:

HR - 105bpm
BP - 95/60mmHg
RR - 26rpm
O2 Sats - 99%

You attach a CTG transducer and the trace shows a baseline rate of 120bpm, variability of 4bpm, with no accelerations but late decelerations occurring in the majority of contractions.

What is the most likely cause of this clinical picture?

A. Bleeding from vasa praevia, causing rapid exsanguination and hypoxia of the fetus
B. Bleeding from a disturbed placenta praevia, causing hypoxia of the fetus
C. Separation of the placenta from the endometrium, and bleeding into the space between them
D. Inflammation and rupture of the appendix
E. Inflammation of the gallbladder secondary to cholestasis of pregnancy

A

C. Separation of the placenta from the endometrium, and bleeding into the space between them

This is a history of a concealed placental abruption: the placenta detaches from the endometrium and begins to bleed into the space between them. The blood does not exit the uterus until the bleeding becomes very severe, so a concealed abruption does not present with bleeding. This occurs in around 20% of abruptions, so the absence of bleeding in abdominal pain in pregnancy is not considered reassuring.

The clues in the history are the tense woody abdomen, the haemodynamic instability, and the pain, along with the CTG suggestive of fetal distress/hypoxia (reduced variability, late declerations). Given the severity of the abruption, this woman would need a C-section, despite the age of the fetus.

How well did you know this?
1
Not at all
2
3
4
5
Perfectly
50
Q

A 22 year old woman is referred to gynaecology with increasingly heavy and painful periods over the last 6 months. Biochemical tests reveal a microcytic anaemia, but normal TFTs, B12, and folate. After thorough investigation these symptoms are attributed to dysfunctional uterine bleeding. She has no PMHx, but has a significant FMHx of VTE. The patient is extremely distressed by the bleeding, and desperately wants to not have to worry about bleeding more than anything.

Which of the following treatments would be best for this patient?

A. Implant a Mirena coil
B. Prescribe tranexamic acid and mefenamic acid
C. Supplementation of iron, B12, and folate to correct the anaemia
D. Ask her to trial aspirin +/- paracetamol for a week then return
E. Prescribe the COCP and advise continuous use

A

B. Prescribe tranexamic acid and mefenamic acid

Tranexamic acid antagonises lysine binding sites on plasminogen, reducing conversion to plasmin and so preventing fibrinolysis. Mefenamic acid is a COX inhibitor that is particularly useful in reducing menstrual pain. When used together they provide synergistic relief from both heavy bleeding and pain, and so are a good option in this patient.

‘A’ is the NICE-recommended first line treatment and is effective at reducing heavy bleeding. However, for the first 6 months a Mirena coil causes constant light bleeding. If the patient can weather this period, then they will get occasional light periods (~1/3 have no periods at all) but in this patient the initial 6 month period is likely to prompt them to either refuse or remove the coil after a few months. The Mirena coil lasts 5 years before it has to be replaced (the copper IUD lasts 10).

‘C’ would be a good step, but the patient is not reporting the symptoms of anaemia, so it isn’t the most pressing concern and the pain and bleeding is affecting her life more.

‘D’ is not a particularly strong option because it seems likely she has tried over the counter medication before, and it will not address the bleeding issue, which she is most concerned about.

‘E’ would be a reasonable option were it not for the significant FMHx of VTE.

How well did you know this?
1
Not at all
2
3
4
5
Perfectly
51
Q

Which of the following statements on complications of the LLETZ is false?

A. There is a 15% chance of recurrence after a LLETZ
B. It may damage the cervix and lead to increased risk of preterm birth
C. It may cause scarring that stenoses the cervix
D. It may make it difficult to monitor the patient with further smears
E. There is a risk of endometritis

A

A. There is a 15% chance of recurrence after a LLETZ

The LLETZ is an effective procedure, and is successful in over 95% of cases:
https://obgyn.onlinelibrary.wiley.com/doi/full/10.1111/j.1471-0528.2000.tb11623.x

How well did you know this?
1
Not at all
2
3
4
5
Perfectly
52
Q

A 27 year old woman presents to her GP with vaginal bleeding unrelated to her periods which sometimes occurs spontaneously, but may also occur after sex. A speculum exam reveals an abnormal looking, bleeding area of the cervix which is then swabbed. Analysis of the swab reveals a squamous cell carcinoma of the cervix. The lesion is biopsied and staged as CIN 3.

Which of the following statements about CIN 3 (cervical intraepithelial neoplasia) is false?

A. It may spontaneously remit
B. It may recur after treatment
C. It is heavily associated with HPV infection
D. It progresses to invasive carcinoma extremely slowly
E. It usually presents in women >50

A

E. It usually presents in women >50

CIN is most common in women in their late 20s/ early 30s, and is associated with HPV infection in 99.7% of cases - infection with HPV is generally considered to be a prerequisite. The transformation zone of the cervix (where squamous epithelium gives way to columnar) is particularly vulnerable to HPV infection, hence it is also more vulnerable to cervical cancer.

CIN is a major cause of death worldwide, but less so in the Western world because of vaccination against HPV. The two main vaccines are Cervarix (bivalent protection against serotypes 16 and 18) and Gardasil (quadravalent protection against 6, 11, 16, 18).

CIN 3 refers to malignancy that occupies >2/3 of the epithelial thickness of the cervix, but has not breached the basement membrane. Progression to full carcinoma (invasion of the basement membrane) is a very slow process and may take decades (one study found the medium progression time to be over 20 years - see below). CIN may recur after treatment, but equally there is a trend in young women to develop dysplastic lesions which then spontaneously remit as the immune system brings them under control.

Link to CIN 3 - cancer progression study:
https://academic.oup.com/aje/article/178/7/1161/211254

NB: This question is taken from a Capsule case

How well did you know this?
1
Not at all
2
3
4
5
Perfectly
53
Q

A 35 year old woman attends her GP after her sister is diagnosed with ovarian cancer. She is concerned for herself and her daughters and wants to learn as much as she can about the disease.

Which of the following statements are true?

A. She should avoid taking the oral contraceptive pill to minimise her risk of developing ovarian cancer
B. The fact she has had children increases her risk of developing ovarian cancer
C. Her sister’s diagnosis significantly increases her risk of developing cancer at some stage
D. Most cases of ovarian carcinoma are caught early whilst they are still confined to the ovary, and prognosis is better in younger women
E. Ovarian cancer typically metastasises to the peritoneum and para-aortic nodes

A

E. Ovarian cancer typically metastasises to the peritoneum and para-aortic nodes

‘C’ is unlikely to be true without a stronger family history, as the majority of women with ovarian cancer have no known risk factors, and genetics are thought to play a part in only 1% of cases. Where there is a genetic basis, it is the strongest risk factor, but the presence of such a risk factor is uncommon.

‘A’ is untrue - taking oral contraceptives has actually been shown to reduce the risk of ovarian cancer, and an important analysis of 45 studies in the Lancet suggested the pill has actually prevented hundreds of thousands of cases (https://www.thelancet.com/article/S0140-6736(08)60167-1/fulltext).

‘B’ is false: having children reduces the risk of multiple cancers, including breast and ovarian. ‘D’ is also false: ovarian cancer is typically caught late due to its subtle and non-specific presentation. This leads to a higher mortality rate than comparable cancers e.g. endometrial or cervical.

How well did you know this?
1
Not at all
2
3
4
5
Perfectly
54
Q

Match the following infections with the appropriate description:

A. Trichomonas vaginalis
B. Bacterial vaginosis
C. Pelvic inflammatory disease
D. Thrush

  1. Frothy yellow/ green discharge, vulvovaginitis, strawberry cervix
  2. Vaginal discharge, cervical excitation, abdominal pain, deep dyspareunia
  3. Vulval pruritis, superficial dyspareunia, cottage cheese discharge
  4. Thin grey/ white discharge with a strong fishy smell, clue cells are seen on microscopy, alkaline vaginal environment
A
  1. Frothy yellow/ green discharge, vulvovaginitis, strawberry cervix - A. Trichomonas vaginalis
  2. Vaginal discharge, cervical excitation, abdominal pain, deep dyspareunia - C. Pelvic inflammatory disease
  3. Vulval pruritis, superficial dyspareunia, cottage cheese discharge - D. Thrush
  4. Thin grey/ white discharge with a strong fishy smell, clue cells are seen on microscopy, alkaline vaginal environment - B. Bacterial vaginosis
How well did you know this?
1
Not at all
2
3
4
5
Perfectly
55
Q

A 34 week pregnant woman presents to the maternal assessment unit with reduced fetal movements. She has noticed for the past two days that the baby has not been moving as much as before, and is very worried.

What is the most appropriate next step?

A. Examine the pregnant abdomen, then perform an USS of the fetus to assess wellbeing and growth
B. Attach a CTG trace to assess fetal wellbeing
C. Attach a CTG trace to assess fetal wellbeing, then ask the mother to record the number of kicks she feels for the next 7 days and come back for a follow-up
D. Assess risk factors for stillbirth and IUGR, then use a handheld doppler to look for a fetal heartbeat
E. Reassure the mother that in 70% of cases of one-off RFM there is no underlying pathology, and tell her to return if the problem persists

A

D. Assess risk factors for stillbirth and IUGR, then use a handheld doppler to look for a fetal heartbeat

The first step when assessing a woman with reduced fetal movements (RFM) is to exclude fetal death. This is done by finding the fetal heartbeat with a handheld doppler.

Once this is done, there should be a general assessment including palpation of the abdomen (to assess size as IUGR is associated with RFM) and a CTG trace to assess fetal wellbeing.

If no abnormalities are detected, the mother should be reassured that 70% of pregnancies featuring one episode of RFM feature no complications. She should be asked to return if it happens again or if she experiences any other symptoms. She should not be asked to keep a formal record of the number of kicks, as this is not a useful indicator of fetal health, and increases maternal anxiety by making the mother fixate on the number of kicks.

How well did you know this?
1
Not at all
2
3
4
5
Perfectly
56
Q

A 37 week pregnant g3p2 woman presents to antenatal clinic: her baby was noted to be in breech position at a previous appointment so she was asked to return at 37 weeks. Examination reveals the baby is still in breech.

How should you counsel the woman on breech birth and external cephalic version?

A. ECV is generally a safe procedure, but you must inform her there is a 5% risk of needing an emergency C-section as a result of complications
B. ECV is a very simple procedure for which she doesn’t need to be given any medications
C. ECV is most often successful, with few babies reverting to breech afterwards
D. ECV is generally painless, though there may be some mild discomfort
E. ECV has a better chance of success in multips, but increases the risk of instrumental delivery or C-section compared to natural cephalic presentation

A

E. ECV has a better chance of success in multips, but increases the risk of instrumental delivery or C-section compared to natural cephalic presentation

‘A’ is incorrect, as there is only a 0.5% risk of complications necessitating an emergency C-section. ‘B’ is incorrect, as she should be given tocolytics (e.g. ritodrine?) to increase the chance of success, and may be offered analgesia if they cannot tolerate the procedure without it.

‘C’ is only slightly wrong - there is a 50-50 chance of success, but it is true that few babies revert to breech after a successful attempt. The chance of success is around 60% in multips and 40% in primips, so overall is considered to be 50% though individual factors affect this considerably. ‘D’ is wrong - it is uncomfortable and may well be painful, so ‘D’ is misleading.

There is no absolute consensus on ECV contraindications, but the following are usually considered: multiple pregnancy, Rh isoimmunisation, APH within the last week, abnormal fetal monitoring, and rupture of membranes

How well did you know this?
1
Not at all
2
3
4
5
Perfectly
57
Q

An emergency call goes out from a room where a 37+3 week pregnant woman is in labour. The registrar arrives and is informed by the midwife that the patient has been in the latent first stage of labour for two hours, and a few minutes ago her membranes ruptured. Shortly after ROM the patient became rapidly SOB, then became drowsy and lost consciousness. A quick A-E approach shows saturations of 91% and crackles in both lungs on auscultation, a HR of 140, and a of BP 80/40.

What is the most likely diagnosis?

A. Myocardial infarction
B. Pulmonary embolism
C. Amniotic fluid embolism
D. Cerebrovascular accident
E. Haemorrhage
A

C. Amniotic fluid embolism

Amniotic fluid embolism is a rare complication of pregnancy, usually occurring during labour, where amniotic fluid enters the maternal circulation causing an anaphylactoid reaction and rapid circulatory collapse. This classically occurs when the membranes rupture, and though it is rare (~1 in 50,000 pregnancies) it is so lethal that it is still a significant cause of maternal mortality and morbidity.

The first sign of amniotic fluid embolism is usually symptoms of hypotension and circulatory collapse with rapid acute heart failure, quickly followed by pulmonary oedema, ARDS, and DIC.

The speed of deterioration along with the bilateral crackles on auscultation make this most likely to be an amniotic fluid embolism, though ‘A’, ‘B’, ‘D’, and ‘E’ are important differentials.

How well did you know this?
1
Not at all
2
3
4
5
Perfectly
58
Q

A 21 year old woman attends an emergency appointment with her GP. She overslept and was unable to take her ‘Micronor’ traditional POP at 7 a.m. as she usually does, and it is now 2 p.m.

How should the GP advise her?

A. Take her pill now then take the next one at the usual time, and use condoms till she has been taking the POP for 48 hours
B. To take her pill now, and not to worry as 1 missed pill does not affect the efficacy
C. To take her pill now, and to take levonorgesterel emergency contraception
D. To take her pill now, and not to worry because she is still within the 12 hour window
E. To take her pill now, and not take the regular 1 week break

A

A. Take her pill now then take the next one at the usual time, and use condoms till she has been taking the POP for 48 hours

Micronor is a brand of progesterone-only pill (POP) that must be taken in the same 3 hour window every day. POPs are taken continuously and do not have a week off (so ‘E’ is wrong).

‘B’ would be true for the COCP when missing one pill does not matter, but for the POP barrier contraception should be used until there is 48 hours worth of coverage. ‘C’ is incorrect because there is nothing to suggest emergency contraception is necessary here. ‘D’ would be correct if she were taking Cerazette which has a 12 hour window in which to take it, but Micronor only has a 3 hour window so she has missed the pill. ‘E’ is incorrect because there is no such break with the POP.

How well did you know this?
1
Not at all
2
3
4
5
Perfectly
59
Q

A 36 year old woman presents to her GP with fears she may be pregnant. She had unprotected sex 4 days (96 hours) ago and is extremely worried that it is too late to do anything to prevent pregnancy.

What is the most appropriate management for this patient?

A. Perform a serum hCG, and offer a medical TOP depending on the result
B. Offer insertion of the copper IUD
C. Perform a urinary pregnancy test, and offer levonorgesterel depending on the result
D. Prescribe levonorgesterel and tell her to take a urinary pregnancy test in 1 week
E. Prescribe the COCP and tell her to take a urinary pregnancy test in 1 week

A

B. Offer insertion of the copper IUD

The copper IUD is effective up to 5 days after either sexual intercourse, or the day of ovulation (whichever is latest). The copper IUD is the most effective form of emergency contraception and prevents implantation of the embryo (making it ethically dicey for some people). Though an excellent form of emergency contraception, the copper coil is not ideal as a LARC because it makes womens’ periods heavier and more painful.

NB: If the woman is having regular menstrual cycles, the day of ovulation will reliably be 14 days from the first day of menstruation.

How well did you know this?
1
Not at all
2
3
4
5
Perfectly
60
Q

A 17 year old girl presents to her GP with concerns she may be pregnant. She has been taking the COCP but has lost track of the days and thinks she has missed the first two pills of her first week. She had unprotected sex with her boyfriend on the last day of her week off (3 days ago - 72 hours).

Which of the following is the best option?

A. Tell her to take the COCP as normal and abstain from sexual intercourse for at least 7 days
B. Tell her to take the COCP as normal and practice barrier contraception for at least 7 days
C. Tell her to take the last missed pill plus the pill for that day, reassure her that missing two days is not significant
D. Tell her to take the last missed pill plus the pill for that day and practice barrier contraception for at least 7 days
E. Tell her to take the COCP as normal, offer emergency contraception, and advise her to use barrier contraception for 7 days

A

E. Tell her to take the COCP as normal, offer emergency contraception, and advise her to use barrier contraception for 7 days

Missing one COCP dose is not concerning, the patient should be advised to take the missed pill immediately and then take the pill as they would normally, even if that means taking two pills in one day. If two pills have been missed (at least 48 hours since the last pill) then the next step depends on which pill was missed.

If the missed pill was in the first week, then the woman will need to use barrier contraception for a week and take emergency contraception if she has had unprotected sex in the preceding 7 days.

If the missed pills were in the 2nd or 3rd week there is less concern: barrier contraception may be used if the woman is worried, but emergency contraception won’t be necessary as long as the rest of the cycle has been taken correctly.

How well did you know this?
1
Not at all
2
3
4
5
Perfectly
61
Q

Match each form of contraception with the correct statement:

A. Is the most effective form of contraception listed (according to Pearl index)
B. Prevents implantation of the embryo as emergency contraception
C. Carries a risk of osteoporosis
D. Carries the highest risk of VTE
E. Lasts for 5 years before it must be replaced
F. A progesterone receptor modulator that may be given as a tablet up to 5 days after unprotected sex as emergency contraception
G. Is taken continuously with no breaks
H. The active ingredient of the Mirena coil which may also be used for emergency contraception

  1. Mini-pill
  2. COCP
  3. Mirena coil
  4. Copper coil
  5. Levonorgesterel
  6. Ulipristal
  7. Depo-provera (progesterone depot injection)
  8. Progestogen implant
A

A. Is the most effective form of contraception listed (according to Pearl index) - 8. Progestogen implant
B. Prevents implantation of the embryo as emergency contraception - 4. Copper coil
C. Carries a risk of osteoporosis - 7. Depo-provera (progesterone depot injection)
D. Carries the highest risk of VTE - 2. COCP
E. Lasts for 5 years before it must be replaced - 3. Mirena coil
F. A progesterone receptor modulator that may be given as a tablet up to 5 days after unprotected sex as emergency contraception - 6. Ulipristal
G. Is taken continuously with no breaks - 1. Mini-pill
H. The active ingredient of the Mirena coil which may also be used for emergency contraception - 5. Levonorgesterel

How well did you know this?
1
Not at all
2
3
4
5
Perfectly
62
Q

An 18 year old woman visits her GP to discuss methods of contraception, specifically the COCP.

Which of the following statements regarding the COCP is true?

A. The COCP is a better option than the mini-pill in patients with a history of migraine
B. The annual risk of VTE in COCP use is ~0.05-0.1%
C. The COCP will make periods lighter and less painful, but it is important to have them regularly by taking every 4th week off
D. The COCP is relatively contra-indicated in women over 35 if they are smoking >30 per day
E. The COCP can be used post-partum after 21 days in breastfeeding women

A

B. The annual risk of VTE in COCP use is ~0.05-0.1%

The annual risk of VTE in COCP use according to the European Medicines Agency is 5-12 per 10,000. In the non-COCP population this risk is 2 per 10,000. The COCP is generally accepted to increase VTE risk 3-3.5 fold, but the absolute increase in risk is only 0.08%.

‘A’ is incorrect because the COCP is relatively contraindicated in migraine without aura, and is absolutely contraindicated in patients who have had migraines with aura due to an increased risk of stroke. ‘C’ is false, though the standard advice has always been to have a week off, there is not benefit to this unless the woman wants to keep track of her periods to reassure her she is not pregnant.

‘D’ is nearly correct, but the COCP is absolutely contraindicated in women over 35 who are smoking more than 15 per day. ‘E’ is wrong because the COCP is absolutely contraindicated for the first 6 weeks post-partum in breastfeeding mothers as it interferes with breastfeeding.

How well did you know this?
1
Not at all
2
3
4
5
Perfectly
63
Q

A 63 year old woman presents to her GP with a 1 month history of unexplained bloating. She has tried two diet changes to resolve the bloating (cutting out gluten, reducing carbohydrates) but it has not improved. She also describes early satiety at mealtimes and constipation.

What is the most appropriate initial management of this patient?

A. Refer them to a specialist GI unit for further investigation +/- OGD
B. Give specialist dietary advice on high fibre food and water intake, and consider trialling laxatives
C. Arrange an USS
D. Measure her serum Ca125 level
E. Take baseline bloods including FBC, U&Es, LFTs, and INR, and take a stool sample

A

D. Measure her serum Ca125 level

This history could well be describing IBS, however new onset IBS in a woman of this age is rare, and all presentations of IBS-like symptoms in this demographic should be investigated for ovarian cancer. Ideally the GP would take a history focusing on the risk factors for ovarian cancer (nulliparity, family history, BMI, PMHx e.g. endometriosis etc.). However that is not an option here, and the next thing that should be done is a serum Ca-125 measurement. If the level is 35 IU/mL, the woman should be referred for an USS of her abdomen and pelvis. If a mass is found she is then referred to a specialist, if not she may require other investigations or be asked to monitor her symptoms and return if they worsen.

If the Ca-125 and USS suggest ovarian cancer, the disease should be staged in secondary care with a CTAP (CT abdo pelvis) with added chest CT if clinically indicated. Histology or cytology should then be used for tissue diagnosis.

How well did you know this?
1
Not at all
2
3
4
5
Perfectly
64
Q

A 75 year old woman visits her GP regarding a lump she has noticed in her vagina. She also describes a sensation of heaviness and dragging that is particularly noticeable in the evening and when she stands up after sitting for a while. She also describes urinary symptoms including hesitancy, increased frequency. On examination there is a bulging of the upper anterior wall of the vagina.

What is the most likely cause of the mass?

A. Apical prolapse
B. Gynaecological malignancy, exploration and excision
C. Enterocoele
D. Cystourethrocoele
E. Rectocoele
A

D. Cystourethrocoele

How well did you know this?
1
Not at all
2
3
4
5
Perfectly
65
Q

A 38-year-old woman presents to her GP with difficulty conceiving. She and her partner have been having regular unprotected sex (once every 2-3 days) for a year and a half. Her periods have been coming irregularly, her cycle is 27-33 days long, and she usually bleeds for ~5 days. The husband’s sperm has already undergone analysis and his count and morphology are normal.

How should this couple be managed?

A. Order a hysterosalpingogram along with day 2 LH and FSH
B. They should be told to continue trying and that if they have not conceived after 2 years of unprotected regular sex they can be seen by a sub-fertility specialist
C. Measure day 21 progesterone and day 2 LH and FSH
D. Educate the couple in the use of temperature-based ovulation predictors to be trialled for 3 months
E. Measure day 2 LH, FSH, and oestradiol, plus AMH

A

C. Measure day 21 progesterone and day 2 LH and FSH

Mid luteal progesterone should be measured to confirm ovulation, though this will be challenging in this patient due to the irregular cycle. In this case the irregular cycle is the indication for measuring gondadotropins.

Other tests would also probably be carried out (e.g. hysterosalpingiogram, AMH, antral follicle count), but the ones described here are those specifically indicated by the menstrual irregularity.

How well did you know this?
1
Not at all
2
3
4
5
Perfectly
66
Q

A 39 year old woman visits her GP regarding contraception. She is 36 weeks pregnant and wants to know how she should manage contraception once she has given birth. She has previously used the COCP and was very happy with it. She is booked in for an elective C-section and plans to breastfeed her baby.

What should the GP tell her?

A. She does not have to worry about falling pregnant within the 6 weeks following birth, but should begin taking the COCP in the 5th week to cover herself
B. She will need to use barrier contraception as soon as she begins to have sex again, and should take a pregnancy test if she has unprotected intercourse at any time post-partum
C. She will not fall pregnant within the first 21 days, then afterwards should use contraception, but not the COCP
D. If she wants an IUD inserted, she will have to wait till 4 weeks post-partum
E. She will not fall pregnant whilst breast-feeding due to the high levels of prolactin in her blood

A

C. She will not fall pregnant within the first 21 days, then afterwards should use contraception, but not the COCP

Women will not fall pregnant within the first 21 days post-partum, but afterwards will need to use contraception. This should be planned for before birth, and can be initiated soon after birth; an IUD can be inserted within 48 hours post-partum, though outside of this window the woman will have to wait 4 weeks (hence ‘D’ is almost right but not quite).

‘A’ is wrong because not only can a woman fall pregnant within the 6 weeks post-partum, she should not take the COCP as it interferes with milk production and so may cause problems with breastfeeding. ‘B’ is wrong because there is a 21 day ‘safe period’. ‘E’ is wrong.

How well did you know this?
1
Not at all
2
3
4
5
Perfectly
67
Q

A 35 year old woman currently in labour at 38 weeks (having declined C-section) begins to experience difficulties. She is obese and a known type II diabetic prior to pregnancy. The midwife calls for help as the head has been delivered but the shoulders are stuck. The labour ward team rush in and first fold the woman’s legs up towards her and apply suprapubic pressure to try and deliver the shoulders, but this fails.

What should be the next step in this patient’s management?

A. Give uterotonics and attempt to deliver the baby’s posterior arm
B. Switch the mother’s position so that she is on all fours, then repeat the previous manoeuvres
C. Perform a Zavanelli manoeuvre to allow the woman to be rushed to theatre for C-section
D. Perform an episiotomy and attempt internal rotational manoeuvres (Wood’s screw)
E. Perform an episiotomy and repeat the suprapubic pressure, then perform a symphysiotomy if it fails

A

D. Perform an episiotomy and attempt internal rotational manoeuvres (Wood’s)

Shoulder dystocia is when the baby’s shoulders become stuck after delivery of the head. It is an obstetric emergency and in these cases there is only minutes in which to act to deliver the baby before brain damage occurs. There is no agreed-upon time limit before brain damage in dystocia, though limiting the time to 5 minutes or less carries a very low risk of hypoxic encephalopathy. There are known risk factors for dystocia including macrosomia, maternal diabetes and obesity, previous dystocia in labour, and induction/ augmentation of labour but there is no good way to predict dystocia and most cases are considered unpredictable.

If the baby is delivered in a way that puts excess traction on the neck, they are at risk of brachial plexus injury, specifically Erb’s palsy (C5, C6 nerve root injury). If the baby is not delivered rapidly they are at risk of brain damage.

Upon suspicion of shoulder dystocia the midwife or doctor should immediately call for help and position the mother lying flat near the edge of the bed. They then implement McRobert’s manoeuvre and apply suprapubic pressure which resolves the dystocia in 90% of cases. If this fails the patient will need an episiotomy to allow a hand into the vagina to attempt an internal rotation manoeuvre (Wood’s screw manoeuvre) and to deliver the posterior arm. If all these fail they may be repeated with the woman on all fours, or the consultant obstetrician may decide on a symphysiotomy (division of the maternal pubic symphysis), cleidotomy (division of the baby’s clavicle), or Zavanelli manoeuvre (pushing the baby back up through the birth canal, then immediately performing a Caesarean section)

See appendix 2:
https://www.rcog.org.uk/globalassets/documents/guidelines/gtg_42.pdf

How well did you know this?
1
Not at all
2
3
4
5
Perfectly
68
Q

A woman in established labour develops a pathological CTG trace, with late decelerations present in the majority of contractions for the past 35 minutes. The obstetric registrar views the trace and concludes there is evidence of fetal distress. She is 40 weeks pregnant and is being induced with an oxytocin infusion. She is contracting at a rate of 5 in 10.

What is the most appropriate next step in this woman’s management?

A. Examine the abdomen, vagina, and check her observations, then move her into the left lateral position and wait 5 minutes to see if that resolves the fetal distress
B. Examine the abdomen and vagina, and if feasible attempt to expedite delivery using forceps
C. Move her into the left lateral position and stop the oxytocin infusion
D. Move her into the left lateral position and stop the oxytocin infusion, examine the abdomen, vagina, and check her observations
E. Attempt to expedite delivery with instruments, and if that fails proceed to C-section

A

D. Move her into the left lateral position and stop the oxytocin infusion, examine the abdomen, vagina, and check her observations

Fetal distress usually refers to signs of fetal hypoxia +/- acidosis, of which late declarations is one. In this case it seems likely that the induction of labour with oxytocin has lead to over-stimulation of the uterus which has caused the fetal hypoxia. Over-stimulation is defined as having a series of contractions each lasting 2 minutes or more, or having 5 or more contractions in 10 minutes.

The most obvious step is to stop the oxytocin infusion, and turning the mother into the left lateral position to relieve pressure on the maternal inferior vena cava and aorta. After that the abdomen and vagina should be examined to look for cord prolapse, presence of liquor, and to examine the tonicity of the uterus. Maternal observations are also important as maternal hypotension an compromise fetal blood supply.

Fetal blood sampling may be used to assess the well-being of the child, and a pH of less than 7.2 is often used as the cutoff for expediting delivery urgently (though Impey suggests this is unnecessarily conservative).

How well did you know this?
1
Not at all
2
3
4
5
Perfectly
69
Q

A 42 year old woman attends her first booking appointment for pregnancy having had her last menstrual period 9 weeks ago. The appointment reveals several risk factors: she has chronic renal impairment and smokes 12 cigarettes per day despite past efforts to quit. She agrees to engage with services to help her quit smoking, and is determined to continue with this pregnancy.

Which is the best next step in the management of this woman’s pregnancy?

A. Book her in for a uterine artery Doppler scan at 20-24 weeks
B. Assess her every 1-2 weeks using abdominal palpation and CTG monitoring
C. Refer her for serial uterine artery Doppler scans and USS for fetal size from 26-28 weeks onwards
D. Book her in for reassessment at the start of the third trimester with a uterine artery Doppler scan and an USS to assess fetal well-being and size
E. Prescribe low does (75mg) aspirin from 16 weeks and reassess her pregnancy at 20 weeks with USS to assess fetal well-being and size

A

C. Refer her for serial uterine artery Doppler scans and USS for fetal size from 26-28 weeks onwards

The chronic renal impairment and smoking of 11 or more cigarettes per day gives this woman a high risk of having a small for gestational age (SGA) baby. There is no way to boost the growth of an SGA baby other than by minimising risk factors, and the only initial management is to monitor the pregnancy.

If there are one or more major risk factors, the mother should be sent for serial assessment of fetal size on USS and uterine artery Doppler scans beginning at 26-28 weeks. If there are 3 or more minor risk factors, the mother should be sent for a uterine artery Doppler at 20-24 weeks. If the 20-24 week Doppler is abnormal, the woman should be referred for serial assessment of fetal size on USS and uterine artery Doppler scans beginning at 26-28 weeks.

Depending on the results of the Doppler scans and the growth of the baby, the obstetrician may recommend delivering the baby early especially if there is static growth. In this case the mother would require steroids to mature the baby’s lungs, and there should be adequate planning for complications depending on the gestational age of the baby when it is delivered.

The RCOG guidelines are very helpful for this area, and there is an investigation flowchart and table of risk factor sin the appendix:
https://www.rcog.org.uk/globalassets/documents/guidelines/gtg_31.pdf

How well did you know this?
1
Not at all
2
3
4
5
Perfectly
70
Q

A 50 year old woman attends her GP to discuss the long-term implications of the menopause. She has not had a period in 14 months and has been experiencing mood swings, hot flushes, insomnia, and a loss of libido combined with vaginal dryness. The insomnia has been affecting her ability to function during the day, and the sexual symptoms are impacting her relationship with her partner. She asks her GP about the benefits and risks of starting HRT.

How should the GP counsel her?

A. Taking a progestogen alongside oestrogen reduces the risk of breast cancer
B. Though her testosterone levels will also drop, it is an unimportant and scarce hormone in women and supplementing it would not benefit her
C. Her risk of osteoporosis will increase post-menopause, but can be offset with vitamin D supplementation if she wants to avoid hormone use
D. Vaginal oestrogen is a promising option for her, as it will improve her libido and vaginal dryness, as well as her hot flushes
E. An oestrogen-only patch would not increase her risk of VTE

A

E. An oestrogen-only patch would not increase her risk of VTE

Menopause has two main functions: control of menopausal symptoms, and improving long-term health. The former reason is the most common indication, as the symptoms of insomnia, hot flushes, irritability/ mood swings, reduced libido, and vaginal dryness can be very distressing.

HRT can be taken in many different preparations, and may consist of oestrogen alone, or there may be progesterone added. The decision to add a progestogen rests on whether the woman still has a womb, as unopposed oestrogen therapy increases the risk of endometrial cancer. However if the woman does not have a womb, lone oestrogen is the preferred choice, because progestogens increase the risk of breast cancer, whereas lone oestrogen therapy has little or no effect on breast cancer risk, hence ‘A’ is wrong.

‘B’ is incorrect because not only are testosterone levels over 100 times higher in pre-menopausal women than oestrogen levels, but testosterone supplementation can be helpful in restoring libido.

‘C’ is correct in that her risk of osteoporosis will increase, but vitamin D supplementation will do nothing to prevent this; don’t confuse osteoporosis with osteomalacia.

‘D’ is true in that it would help her vaginal dryness, but a vaginal preparation will not improve her libido or her hot flushes. Vaginal oestrogen’s major advantage is its lack of systemic side-effects, and it does not increase the risk of cancer or VTE. However this lack of systemic effect also means it will not improve symptoms of the menopause outside of the vagina. HRT patches (E) are a way of taking HRT without any increased risk of VTE, whilst still addressing the symptoms of menopause.

More information on HRT:
https://www.nice.org.uk/guidance/ng23/ifp/chapter/Benefits-and-risks-of-HRT

How well did you know this?
1
Not at all
2
3
4
5
Perfectly
71
Q

Which of the following is considered a minor, not a major risk factor for an SGA baby?

A. Renal impairment
B. BMI <20
C. Unexplained ante-partum haemorrhage during the pregnancy
D. Daily vigorous exercise while pregnant
E. Cocaine use while pregnant

A

B. BMI <20

All pregnant women should be screened for risk factors for a small for gestational age (SGA) baby at their booking appointment (8-10 weeks). There are many risk factors which have been divided into major and minor by RCOG depending on the odds ratio with respect to having an SGA baby based on a meta analysis study.

How well did you know this?
1
Not at all
2
3
4
5
Perfectly
72
Q

The obstetric consultant is called to see a 37 week pregnant primip mother in the second stage of labour who has not progressed for the last 3 hours. She has an epidural anaesthesia tube in situ that has been working well, she is 10cm dilated but has seemingly not progressed in the second stage of labour and is very fatigued. The fetal head is 1/5 palpable and the vertex is at station +1cm. Operative delivery is decided to be the best course of action and the woman is consented.

How would this operative delivery be classified?

A. Outlet
B. Low
C. Mid
D. High
E. This would require Caesarean section
A

C. Mid

Operative delivery is indicated to shorten the second stage of labour in a mother at risk of complications, to resolve fetal distress, or to deliver a baby where progress of labour has halted.

Operative delivery should only be used if the fetal head is 1/5 or less palpable, and the leading part of the head has at least reached the ischial spines. This of course means that operative delivery is only possible once the cervix is fully dilated and the baby has descended into the pelvis.

Operative vaginal deliveries are classified as below:

Outlet:
Fetal scalp visible without separating the labia
Fetal skull has reached the pelvic floor
Fetal head is at or on the perineum

Low:
Leading point of the skull (not caput) is at station plus 2 cm or more and not on the pelvic floor

Mid:
Fetal head is no more than 1/5th palpable per abdomen
Leading point of the skull is above station plus 2 cm but not above the ischial spines

High:
Not included in the classification as operative vaginal delivery is not recommended in this situation where the head is 2/5th or more palpable abdominally and the presenting part is above the level of the ischial spines

GTG 26 on operative delivery:
https://www.rcog.org.uk/globalassets/documents/guidelines/gtg_26.pdf

How well did you know this?
1
Not at all
2
3
4
5
Perfectly
73
Q

A 15 week pregnant woman visits hospital for amniocentesis to determine her baby’s risk of Down syndrome, as her combined test indicated an increased risk of Down syndrome.

Which of the following should this woman be offered after her amniocentesis?

A. No anti-D prophylaxis is required
B. 500 IU anti-D prophylaxis
C. Standard routine anti-D prophylaxis either as a single dose at 29 weeks, or as two doses at 28 and 34 weeks
D. 500 IU anti-D prophylaxis and an FMH test
E. 250 IU anti-D prophylaxis

A

E. 250 IU anti-D prophylaxis

Sensitising event under 12 weeks = no action needed
Special sensitising event* under 12 weeks = min. 250 IU
Sensitising events 12-20 weeks = min. 250 IU
Sensitising events 20+ weeks = min. 500 IU + FMH test** (includes labour***)
Routinely give 1 dose (1500 IU) at 28 weeks or 2 doses (min. 500 IU each) at 28 and 34 weeks

  • Special sensitising events are molar pregnancy, ectopic pregnancy, termination of pregnancy, or uterine bleeding that is heavy, repeated, or painful
  • *Feto-maternal haemorrhage test is also known as a Kleihauer–Betke test and is used to quantify the amount of fetal haemoglobin that enters the mother’s bloodstream
  • **Anti-D prophylaxis after birth is based on typing of baby’s cord blood

NB: This is obviously only necessary in a Rh- non-sensitised woman, and a woman’s blood type will be tested for at her booking appointment (before 10 weeks)

Guidelines:
https://onlinelibrary.wiley.com/doi/full/10.1111/tme.12091

How well did you know this?
1
Not at all
2
3
4
5
Perfectly
74
Q

A 25 week pregnant woman presents to her GP with suprapubic pain and dysuria. The symptoms began a day ago and have been accompanied by small amounts of blood. She is otherwise well with normal observations.

How should this patient be managed?

A. Take an MSU for culture and sensitivities, then prescribe antibiotics based on the results when available
B. Tell her to go to A&E urgently
C. Take an MSU for culture and sensitivities, perform a urine dipstick, and prescribe Nitrofurantoin 100mg BDS for 7 days
D. Encourage her to remain hydrated, perform a urine dipstick test, and prescribe Trimethoprim 200mg BDS for 14 days
E. Encourage her to rest and stay hydrated, recommend or prescribe analgesia, but counsel her that avoiding antibiotics is preferable and that she should return if the symptoms persist in 3 days

A

C. Take an MSU, perform a urine dipstick, and prescribe Nitrofurantoin 100mg BDS for 7 days

Management of a UTI in pregnancy is similar to management in a non-pregnant woman. Nitrofurantoin is first line and safe to give in pregnancy, though NICE recommends it is avoided near term. Second line is Amoxicillin (depending on culture sensitivities) or Cefalexin.

If the patient shows signs of systemic illness or more severe pain in their side, they may need to attend secondary care and be treated for pyelonephritis.

How well did you know this?
1
Not at all
2
3
4
5
Perfectly
75
Q

A 24 year old woman presents to her GP with a 9 week history of amenorrhea and a 2 week history of nausea and vomiting. A urinary pregnancy test done in the clinic is positive, but when she attends her dating scan a few weeks later the pregnancy cannot be visualised in the uterus. An adenexal mass is visualised within the left fallopian tube which moves separately to the ovary; this is diagnosed as a clear ectopic pregnancy. A serum bhCG is taken and the level is 1800 IU/L. Her observations are stable and she is reporting no other symptoms. She is asked to visit hospital two days later when the bhCG is repeated and is 1870 IU/L.

How should this patient be managed?

A. Employ a watchful waiting approach as the ectopic pregnancy may self-resolve, take serial beta-hCG measurements on day 2, 4, and 7
B. Refer her for an urgent laparoscopy +/- salpingotomy
C. Refer her for emergency laparoscopy and saplingectomy
D. Offer a choice of expectant management or a single I.M. dose of 5mg/square metre methotrexate
E. Offer a choice of laparoscopy or a single I.M. dose of 5mg/square metre methotrexate

A

E. Offer a choice of laparoscopy or a single I.M. dose of 5mg/m^2 methotrexate

Though this woman has been diagnosed with an ectopic pregnancy - a very serious and potentially dangerous condition - she is currently asymptomatic and haemodynamically stable. A more acute picture would necessitate emergency surgical management, but this picture means that treatment does not need to be rushed and does not necessarily need to be surgical.

In these cases beta hCG is used to guide treatment:
<1000 IU/L: expectant management
<1500 IU/L: patient’s choice between methotrextate and expectant management
1500-5000 IU/L: patient’s choice between methotrexate and surgery
>5000 IU/L: surgery
NB: The ectopic pregnancy must not be larger than 35mm, there must be no visible heartbeat, and no pain or haemodynamic instability, otherwise surgery is indicated

‘A’ would only be appropriate for a beta-hCG of less than 1500 IU/L, though it is correct in that beta-hCG must be re-measured on day 2, 4, and 7. If the value drops by 15% by day 7, then monitoring is continued until it is less than 20IU/L. If the value does not drop by 15% by day 7, then senior advice is sought.

How well did you know this?
1
Not at all
2
3
4
5
Perfectly
76
Q

The obstetric consultant is called to see a 37 week pregnant g3p2 mother in the second stage of labour who has not progressed for the last 2 hours. She has an epidural anaesthesia tube in situ that has been working well, she is 10cm dilated but has seemingly not progressed in the second stage of labour and is very fatigued. The fetal head is 3/5 palpable and the vertex is at station -1cm.

How should this patient be managed?

A. Carry out fetal blood sampling and use the pH in conjunction with CTG tracing to guide management
B. Expedite delivery using forceps
C. Begin an oxytocin infusion and reassess in 1 hour, monitoring obs every 15 minutes
D. Expedite delivery using ventouse
E. Caesarean section

A

E. Caesarean section

Seeing as the fetal head is more than 1/5 palpable, this baby is not suitable for operative delivery. At 3/5 palpable this baby’s head is not even engaged in the pelvis, which in combination with failure to progress means this woman will need a C-section.

How well did you know this?
1
Not at all
2
3
4
5
Perfectly
77
Q

A 56 year old woman presents to her GP with a 2 week history of vaginal bleeding. Her last menstrual period was when she was 50, and she has been taking HRT since symptoms of the menopause began. She denies any pain, other discharge, or fever, and states the bleeding happens most days and does not appear to have a pattern or be a consequence of sex.

How should this woman be managed?

A. Refer to secondary care for a CTAP
B. Perform a speculum and bimanual exam, and let the findings guide further investigations and management
C. Perform an USS in primary care and act based on the results
D. Urgent 2 week suspected cancer referral
E. Make a routine referral to a specialist gynaecologist

A

D. Urgent 2 week suspected cancer referral

Post-menopausal bleeding should essentially always trigger an urgent referral because of the risk of endometrial cancer.

How well did you know this?
1
Not at all
2
3
4
5
Perfectly
78
Q

A 36 week pregnant woman attends an antenatal care appointment, and the doctor palpates her abdomen and determines her baby is currently in the breech position. She is offered ECV but is not keen and wants to know more about what will happen if her baby is delivered breech. She hopes to avoid any major surgery.

Which of the following is not true regarding breech birth?

A. If she opts for a planned C-section, that will not increase the risk of complications of vaginal birth in a subsequent pregnancy
B. The risk of perinatal mortality for planned breech delivery is 4 times higher than in C-section, but still low
C. There is a 40% chance the mother will need an emergency Caesarean - the most high risk method of delivery
D. Breech presentation at term occurs in 3-4% of pregnancies
E. C section is recommended in the presence of certain risk factors e.g. high or low fetal weight, Footling presentation, evidence of fetal distress

A

A. If she opts for a planned C-section, that will not increase the risk of complications of vaginal birth in a subsequent pregnancy

https://obgyn.onlinelibrary.wiley.com/doi/epdf/10.1111/1471-0528.14465

How well did you know this?
1
Not at all
2
3
4
5
Perfectly
79
Q

At what point is the fetal head considered to be ‘engaged’?

A. When the widest point of the presenting part is in line with the ischial spines
B. When 3/5 or less of the head is palpable abdominally
C. When the widest diameter of the presenting part passes through the pelvic inlet
D. When the presenting part reaches a positive station
E. When the presenting part reaches station +5cm

A

C. When the widest diameter of the presenting part passes through the pelvic inlet

Engagement usually happens at 34-35 weeks and refers to when the widest point of the presenting part passes the pelvic inlet. At this point the head will be a maximum of 2/5 palpable by definition, and generally the vertex will be level with the ischial spines, though obviously this depends on the size of the baby and the mother’s pelvis.

How well did you know this?
1
Not at all
2
3
4
5
Perfectly
80
Q

An obstetric consultant visits a new mother who gave birth 5 days ago on the postnatal ward, and while speaking with her notices her mood seems low. The mother confirms that she feels somewhat depressed and tired, and guilty as she does not feel how she thought she would as a new mother.

How should this mother be managed?

A. Trial fluoxetine
B. Take a history to assess for psychiatric issues, but reassure her this is very likely ‘baby blues’ which happens in half of all women after childbirth and will soon pass
C. Take a history to assess for psychiatric issues, and try to assess the mother’s symptoms more formally e.g. with the Edinburgh Postnatal Depression Scale
D. Take a brief focused history, then contact Liason Psychiatry and ask them to come and assess the patient
E. Take a history to assess for psychiatric issues, then offer a trial of outpatient therapy to see whether it alleviates her symptoms

A

B. Take a history to assess for psychiatric issues, but reassure her this is very likely ‘baby blues’ which happens in half of all women after childbirth and will soon pass

The ‘baby blues’ affect 50% of all women during the first week after childbirth and only persist a few days. It causes low mood, irritability, anxiety, and labile mood. Whilst guilt is not a classic feature, it is understandable why this mother feels guilty that she feels this way when societal expectations tell her she should be ecstatic.

Postnatal depression in 10%, 70% risk of depression in later life.

How well did you know this?
1
Not at all
2
3
4
5
Perfectly
81
Q

For each of the following side effects, decide whether they are more associated with progesterone or oestrogen-based contraception:

A. Fluid retention
B. Osteoporosis
C. Reduced libido
D. Hypertension
E. Headaches
A
A. Fluid retention - progesterone
B. Osteoporosis - progesterone (specifically depot-Provera)
C. Reduced libido - progesterone
D. Hypertension - oestrogen
E. Headaches - oestrogen
82
Q

According to UKMEC, in which of the following is the COCP absolutely contraindicated?

A. A woman who gave birth 4 weeks ago and is bottle feeding her baby
B. A 37 year old woman who smokes 10 cigarettes per day
C. A woman with a BMI of 38
D. A woman with blood pressure of 165/110
E. A woman whose mother had an idiopathic VTE aged 36

A

D. A woman with blood pressure of 165/110

All other options are rated ‘3’ by UKMEC i.e. the risks most likely outweigh the benefits, but hypertension of over 160/100 is ‘4’ i.e. absolutely contraindicated.

Other absolute contraindications include:
Vascular disease
Previous or current VTE
Migraine with aura
Being within the puerperium and breastfeeding
Being within the first 3 weeks of the puerperium and having risk factors (e.g. obesity, PPH, pre-eclampsia, smoking)
Being over 35 and smoking 15 or more cigarettes per day

83
Q

A 30 week pregnant woman is brought to A&E with vomiting, jaundice, and epigastric pain. She is febrile, seems confused, and has a purpuric rash on her legs. A rapidly taken set of bloods show the following:

Hypoglycaemia
A prolonged PT
A normal haemoglobin
Thrombocytopenia
Elevated transaminase enzymes
Metabolic acidosis

Her husband is also in attendance and says her pregnancy has been uncomplicated and she has been mostly well, though she has complained of being especially tired and thirsty for the past couple of weeks.

What is the most likely diagnosis?

A. Acute fatty liver of pregnancy
B. Obstetric cholestasis
C. HELLP syndrome
D. Eclampsia
E. Pancreatitis
A

A. Acute fatty liver of pregnancy

This is a history of acute fatty liver of pregnancy - a 1 in 20,000 condition that usually affects women late in the third trimester. Its pathogenesis is not well understood, but is thought to be linked to an underlying mitochondrial disorder affecting fatty acid oxidation.

Fat builds up in the liver causing dysfunction which leads to jaundice (decreased bilirubin clearance, DIC, hypoglycaemia, and metabolic acidosis (reduced lactate clearance). Classic features include epigastric pain, vomiting, and excessive thirst. This is a difficult pathology to separate from the preeclampsia - eclampsia - HELLP conditions and is theorised to be on the same spectrum.

84
Q

A 38 year old woman presents to her GP with a history of incontinence. This has been an issue since the birth of her last child, who was her fourth. She reports leaking of urine when she laughs, coughs, or runs.

Given the likely diagnosis, what is the first step in this patient’s management?

A. Offer bladder wall botulinim injection
B. Refer her to a physio for pelvic floor exercises
C. Offer a 6 week bladder training program
D. Trial use of anti-cholinergics (e.g. Oxybutynin)
E. Evaluate lifestyle factors e.g. BMI, caffeine intake, and fluid intake

A

E. Evaluate lifestyle factors e.g. BMI, caffeine intake, and fluid intake

Lifestyle factors have a significant impact on many diseases, including incontinence. There isn’t much point treating the patient for incontinence if the issue could be solved by altering fluid or caffeine intake. It should also be noted that BMI can contribute to stress incontinence, and if a woman has a BMI >30, she should be advised to lose weight.

If lifestyle modifications fail, behavioural therapy (bladder training) or physiotherapy (pelvic floor training) can be used as first line interventions depending on whether the incontinence is stress, urgency, or mixed (physio for stress, bladder training for urge, either for mixed).

Medications may prove useful in patients with urge incontinence, but only after more conservative measures, and only for urge incontinence which is not described here.

85
Q

How should new mothers diagnosed with diabetes before pregnancy manage their insulin use post-partum?

A. Return to their pre-pregnancy insulin dose in stages over a period of 2 weeks, with careful blood glucose monitoring
B. Return to their pre-pregnancy insulin dose in stages over a period of a week, with careful blood glucose monitoring
C. Continue on the current insulin dose for the duration of the puerperium, and see their specialist at 6 weeks
D. Resume their treatment regimen pre-conception including and medications that were substituted
E. Immediately reduce their insulin and monitor their blood glucose to establish the most appropriate dose

A

E. Immediately reduce their insulin and monitor their blood glucose to establish the most appropriate dose

Insulin requirements for diabetics rapidly return to normal post-partum, so their insulin should be immediately reduced to avoid hypoglycaemia. Any medications discontinued for safety reasons pre-conception should still be avoided in a woman who is breastfeeding.

86
Q

The obstetric registrar is called to a room on the antenatal ward where a pregnant woman has collapsed and is currently on the floor.

What should the doctor do immediately?

A. Call for help, secure the woman’s airway, move her into the left lateral position, and give 100% supplemental oxygen
B. Call for help, secure the woman’s airway, listen for breathing, then begin CPR if indicated
C. Call for help, secure the woman’s airway, listen for breathing, move her into the left lateral position, then begin CPR if indicated
D. Call for help, secure the woman’s airway, give supplemental 100% oxygen, and attach a cardiac monitor
E. Call for help, secure the woman’s airway, protect her cervical spine, and give supplemental 100% oxygen

A

C. Call for help, secure the woman’s airway, listen for breathing and move her into the left lateral position

Management of acute collapse of a pregnant woman is similar to that for a non-pregnant adult, though in pregnant women it is particularly important to protect the airway as there is a greater chance of regurgitation and aspiration. Furthermore the woman should be moved into a left lateral tilt to relieve pressure from the gravid uterus on the Vena Cava and Aorta.

See appendix II:
https://www.rcog.org.uk/globalassets/documents/guidelines/gtg_56.pdf

87
Q

A 39 year old woman who is 31 weeks pregnant with DCDA twins presents to A&E with acute onset generalised, severe abdominal pain. She denies any vaginal bleeding or other symptoms. Her abdomen is tense on palpation and her obs are as follows:

Temp: 37.1 oC
SO2: 99%
RR: 27 bpm
HR: 120 bpm
BP: 88/63 mm/Hg

What is the best next step in this patient’s management?

A. Insert two wide bore cannulas, give fluid resuscitation, take bloods, attach a CTG probe and expedite delivery
B. Make her NBM, establish IV access, insert an NG tube, and send her for an urgent abdominal x-ray
C. Admit her to the labour ward, establish I.V. access, take bloods, give steroids, and observe her
D. Thoroughly assess risk factors for placental abruption, and use this to guide the next step
E. Take a focused history and arrange an urgent USS to establish the location of the placenta

A

A. Insert two wide bore cannulas, give fluid resuscitation, take bloods, attach a CTG probe and expedite delivery

This is a history of placental abruption, albeit concealed one. 1 in 5 abruptions are concealed (no vaginal bleeding) so vaginal bleeding should not be relied upon to spot this diagnosis. The key to this history is the acute onset of severe abdominal pain in a pregnant woman, which should always make you consider abruption.

In this case there are two risk factors: advanced maternal age and a multiple pregnancy, though as many as 70% of cases will occur in supposedly low-risk pregnancies, hence ‘D’ is wrong and dangerous.

‘B’ is the general management of an acute abdomen which is a differential here, but not the foremost. ‘C’ would be a good option if the mother was stable but doctor suspected there was a high chance of premature birth. However in this case the mother is haemodynamically compromised, so delivery must be expedited (this is usually through C-section).

‘E’ includes two important steps in a woman presenting with ante-partum haemorrhage where the placenta position is unknown, but the USS would not come before CTG monitoring to establish the health of the baby, and is not appropriate in an emergency such as this.

APH guidelines:
https://www.rcog.org.uk/globalassets/documents/guidelines/gtg_63.pdf

88
Q

A 10 week pregnant 34 year old woman presents to A&E with sudden onset chest pain and breathlessness. She was previously well with no significant PMHx. A quick assessment reveals no other abnormalities, and no tenderness, swelling, or erythema of her calves. Her basic obs are as follows:

Temp: 38.1
SO2: 92%
RR: 27
HR: 110
BP: 123/87

How should this patient be managed?

A. Give oxygen, admit her, establish I.V. access, and schedule a compression Doppler USS and a V/Q scan
B. Give oxygen, assess her risk factors for a PE, and use a Wells score to determine whether a d-dimer or imaging would be most appropriate
C. Give oxygen, admit her, take baseline bloods, give subcutaneous Clexane, arrange an ECG and CXR
D. Give oxygen, admit her urgently and arrange transfer to a cathlab for embolectomy or thrombolysis
E. Give oxygen, perform a cardiovascular examination, and take bloods for troponins and creatine kinase

A

C. Admit her, take baseline bloods, give subcutaneous Clexane, arrange an ECG and CXR

For a pregnant woman presenting with a clinically suspected PE, the first steps are clinical assessment, baseline bloods to determine whether LMWH dose needs adjustment, give LMWH, and perform an ECG and CXR.

The CXR then guides treatment: if it is abnormal then a CTPA is indicated, but if it is normal then a V/Q scan is preferred. CTPA is not contra-indicated in pregnancy, and the decision whether to use a V/Q scan or CTPA is an individual one influenced by equipment availability, maternal age and PMHx/ FMHx, and clinical factors. The main concern with a CTPA is irradiation of a young woman’s breasts and the subsequent risk of breast cancer. CTPA is actually believed to expose the fetus to less radiation than a V/Q scan, and the increase in risk of breast cancer is very small, but must still be considered.

If a woman presents with a suspected DVT +/- signs of a PE, a compression Duplex USS is performed, and if this is positive no further investigation is needed and they are treated with LMWH.

https://www.rcog.org.uk/globalassets/documents/guidelines/gtg-37b.pdf

89
Q

A 36 year old g2p1 woman is invited for gestational diabetes screening at 24 weeks due to her high BMI. She is asked to fast for 8 hours so a fasting blood glucose test can be done, then she is given a 75g 2 hour OGTT.

Which of the following results would allow her to be diagnosed with gestational diabetes?

A. A fasting blood glucose of 5.8 or more, or an OGTT of 7.6 or more
B. A fasting blood glucose of 5.6 or more, or an OGTT of 11.1 or more
C. An HbA1c of 6.5% or more
D. A fasting blood glucose of 5.6 or more, or an OGTT of 7.8 or more
E. An HbA1c of 7.0% or more

A

D. A fasting blood glucose of 5.6 or more, or an OGTT of 7.8 or more

Gestational diabetes is screened for with an oral glucose tolerance test (OGTT) between 24-28 weeks in those who are identified as at risk in their booking appointment. Risk factors include: Asian, Afro-Caribbean, or Middle Eastern race; BMI over 30; previous macrosomic baby; or FMHx of type II diabetes. Previous pregnancy with gestational diabetes is obviously a big risk factor, and these women are screened much earlier, shortly after their booking test.

It is worth noting that neither random blood glucose nor HbA1c is used to diagnose gestational diabetes, though HbA1c is used to monitor it.

Management of diabetics during pregnancy is complex, with myraid potential complications. Established diabetics face more complications than gestational diabetics, with type I and II faring similarly. Maternal complications include: increased birth trauma and delivery complications (due to macrosomia); increased insulin requirement; hypoglycaemia (from attempts to control tightly); UTIs, wound infection, and endometritis; preeclampsia; worsening of pre-existing ischaemic heart disease; and worsening of diabetic retinopathy.

Management of these patients begins pre-conceptually, where the patient is advised to closely monitor their blood glucose levels. Diabetic prospective mothers are advised to take the higher dose of folic acid (5mg instead of 400mcg) for 3 months before conception up until week 12 of pregnancy; this is because of the increased risk of neural tube defects (there is also increased risk of cardiac defects). Pre-existing diabetics are at high risk of developing preeclampsia during pregnancy, and so should be given 75-150mg aspirin ODS from 12 weeks till the end of their pregnancy.

Retinal function, renal function, signs of preeclampsia, and fetal growth must all be monitored during pregnancy. Labour is more likely to be complicated, so problems like increased birth trauma and shoulder dystocia should be planned for. Post-partum the baby is particularly vulnerable to hypoglycaemia which must be monitored for. Finally, it is important to note that the mother’s insulin requirement will rapidly return towards pre-pregnancy after delivery, and failure to attenuate insulin doses may lead to maternal hypoglycaemia.

Gestational diabetes in diabetes presenting during pregnancy in susceptible women, due to changes in metabolism. It account for ~90% of diabetes in pregnancy, increase the risk of developing type II diabetes later, and is associated with conditions impairing glucose tolerance e.g. PCOS. An OGTT is performed between 24-28 weeks to assess for gestational diabetes, or earlier (shortly after the booking appointment) if the woman has developed it before.

NB: glycosuria in pregnancy is a less significant finding than normal because a pregnant woman’s kidneys have a lower threshold for excreting glucose (normal being serum concentration of 11mmol/L)

90
Q

A 53 year old woman presents to her GP with queries about contraception. Her last menstrual period was 14 months ago, and she has been experiencing fatigue and hot flushes, though does not want HRT. She is asking whether she can now dispense with contraception.

What should the GP tell her?

A. She should continue to use barrier methods until menopausal symptoms dissipate
B. It is safe for her to stop using contraception
C. She should continue using hormonal contraception for 10 more months
D. She still needs contraception, but non-hormonal methods are advised
E. She should take contraception till she is 55

A

B. It is safe for her to stop using contraception

For menopausal women it is recommended to use contraception for 24 months after the last period in under 50’s, and for 12 months in women aged 50 or over.

91
Q

A 12 year old girl is seen in GUM clinic with vaginal discharge indicative of an STI. She admits to being sexually active with a boy - also 12 - for 4 months, and has not been using protection.

How should the GUM doctor proceed?

A. Advise her that she needs her parent’s consent for treatment so she will have to inform them before being given antibiotics
B. Perform a pregnancy test and give her antibiotics if she is deemed competent, and tell her they’ll need to contact the safeguarding lead and social services/ the police
C. Perform a pregnancy test and give her antibiotics if she is deemed competent, then try to persuade her to stop her sexual activity
D. Perform a pregnancy test and give her antibiotics if she is deemed competent, and counsel her on safe sex practices
E. Try to convince her to involve her parents, but give her the antibiotics if she is Gillick competent and

A

B. Perform a pregnancy test and give her antibiotics if she is deemed competent, and tell her they’ll need to contact the safeguarding lead and social services/ the police

Contrary to popular belief, you do not always have to disclose sexual activity in a child under 13 to the police. However you should automatically disclose it unless there is a very good reason not to, and it should at least be discussed with a safeguarding lead and documented.

In this case there is no good reason not to report this relationship, so it should be discussed with a safeguarding lead, then the police and social services should be involved.

With regards to the pregnancy test and giving antibiotics, the girl can consent if deemed Gillick competent. If the girl is not competent, treatment may still proceed based on her best interests as determined by a doctor.

GMC guidelines on consent and sexual activity in children:

https: //www.gmc-uk.org/ethical-guidance/ethical-guidance-for-doctors/0-18-years/making-decisions#paragraph-24
https: //www.gmc-uk.org/ethical-guidance/ethical-guidance-for-doctors/0-18-years/sexual-activity

92
Q

A 12 week pregnant woman presents to her GP asking for a termination of pregnancy.

Which of the following would be the most appropriate method?

A. Mifepristone followed by Misoprostol
B. Vacuum aspiration under general anaesthesia
C. Fetal KCl injection
D. Dilatation and evacuation
E. Misoprostol alone
A

A. Mifepristone followed by Misoprostol

The choice between medical and surgical management of a TOP is mostly up to the patient, though they are generally used at different stages in pregnancy. Mifepristone and Misoprostol can be used in any termination before 24 weeks. Vacuum aspiration may be used up to 16 weeks, but does not require general anaesthesia. Dilatation and evacuation is appropriate for pregnancies of >14 weeks gestation.

24 weeks gestational age is the legal cutoff for abortions generally, but they may be performed all the way up to term if:
The mother’s life is at risk
Continuing the pregnancy poses a serious risk to the mother’s physical or mental health
There is a substantial risk of the baby being handicapped

In such cases the fetus is injected with KCl (potassium chloride).

93
Q

Which of the following is usually increased in a Down syndrome pregnancy?

A. Alpha fetoprotein
B. PAPP-A
C. Beta-hCG
D. Unconjugated oestradiol
E. Nuchal opacity
A

C. Beta-hCG

Unconjugated oestriol, hCG, AFP and Inhibin A are the four hormones used in the ‘quadruple test’ for Down syndrome. The quad test is used if the mother misses the 11-14 week window for the standard Down syndrome screening, or if nuchal translucency cannot be measured. Unconjugated oestriol and AFP are decreased in a Down syndrome pregnancy, but Inhibin A and beta-hCG which are raised.

94
Q

What layers have to be dissected or traversed to perform a C-section?

A

Skin - subcutaneous tissues (Camper’s and Scarper’s) - rectus sheath - rectus abdominis - parietal peritoneum - vesicouterine peritoneum - uterus

95
Q

A 27 year old G1P0 woman presents to the maternal assessment unit with vaginal bleeding. The vaginal bleeding is of small volume and this is the first occurrence. She is 10 weeks pregnant, and has been experiencing a fair amount of nausea and vomiting, though not enough for a diagnosis of hyperemesis.

What would be the most appropriate investigation in this woman?

A. Trans-vaginal USS
B. Serial serum beta-hCG
C. CT pelvis
D. Bimanual exam
E. Serum AFP, PAPP-A, b-hCG, and inhibin A
A

A. Trans-vaginal USS

This is a case history of a molar pregnancy - a subset of gestational trophoblastic disease where the proliferating cells are abnormal and become invasive rather than developing into a viable pregnancy. This results either from a sperm fertilising an empty ovum (no chromosomes - complete mole) or from two sperm fertilising one ovum (partial mole). In both cases the resulting tissue can never become a viable pregnancy and must be removed before it invades and metastasises.

It can only be definitively diagnosed by analysing histology, but suggestive features within the history are: nausea and vomiting, irregular vaginal bleeding, and an abnormally large uterus. The classic finding on USS is of a ‘snowstorm appearance’, and b-hCG can be useful as it is often much higher in molar pregnancy than normal.

Management is with suction and curettage of the pregnancy, or evacuation if needed. The tissue is sent to histology to confirm the diagnosis, and anti-D prophylaxis is given in Rh- women as this is a potentially sensitising event.

96
Q

Which of the following will have the most effect on the symptoms of obstetric cholestasis?

A. Topical emollients
B. Ursodeoxycholic acid
C. S-adenosyl methionine
D. Chlorphenamine
E. Colestyramine
A

B. Ursodeoxycholic acid

Emollients and antihistamines (chlorphenamine) may help a little: emollients will help moisturise damaged skin and the antihistamines will cause sedation and aid sleep. However the only effective agent for symptom control is ursodeoxycholic acid (UDCA).

There is insufficient evidence on S-adenosyl methionine for its use, and colestyramine may improve pruritis but is poorly tolerated.

97
Q

A 30 week pregnant woman attends the maternal assessment unit with abdominal pains. She describes them as 3/10 in terms of pain, they come and go, and they feel like a stitch. She has otherwise been well, and abdominal and vaginal exams are normal.

What would be the most appropriate next step?

A. Give tocolytics
B. Admit for at least 48 hours, prescribe erythromycin and consider steroids
C. Take bloods from the mother including FBC and G&S
D. Perform a CTG and USS of the baby
E. Reassure her and send her home

A

E. Reassure her and send her home

This is describing Braxton-Hicks contractions, involuntary movements of the uterine smooth muscle that do not signal the onset of labour. They are generally mild and more uncomfortable than painful, and are completely normal in the second/ third trimester.

98
Q

A 32 year old woman visits her GP for antenatal counselling. During their discussion the GP assesses the woman’s risk factors for various events during pregnancy, and the woman reveals she had a DVT after a flight to France 4 years ago.

What level of anticoagulation would this woman require during pregnancy?

A. None as long as no underlying thrombophilia was found
B. None as long as there are no current risk factors
C. She will require Clexane from the start of her pregnancy till delivery
D. She will require Clexane from 28 weeks gestation till 6 weeks post-partum
E. She will require Clexane from the start of her pregnancy till 6 weeks post-partum

A

E. She will require Clexane from the start of her pregnancy till 6 weeks post-partum

A previous VTE that is unrelated to major surgery is a massive risk factor and so this woman will need anticoagulation throughout pregnancy and the puerperium.

The decision on whether to anticoagulate a pregnant women, and for how long, is based on the presence of risk factors identified antenatally:

Previous VTE without major surgery - definitely anticoagulate full pregnancy and puerperium
Intermediate risk factor (e.g. cancer, thrombophilia) -probably anticoagulate full pregnancy and puerperium
4 minor risk factors - probably anticoagulate full pregnancy and puerperium
3 minor risk factors - anticoagulate from 28 weeks
2 minor risk factors - anticoagulate for the first 10 days post-partum

NB: There is also a ‘very high risk’ category for people with a previous VTE on a background of thrombophilic disease, or whilst on anticoagulants. These patients get high dose LMWH.

99
Q

A 38 year old primiparous woman visits her GP for her booking appointment. The GP screens her for VTE risk factors and, apart from her age, the only other one they find is her BMI (34).

For what period will this woman need anti-coagulating with Clexane?

A. At least 10 days post-partum, but not during the pregnancy
B. For the entirety of her pregnancy and the puerperium
C. From 28 weeks to term
D. From 20 weeks till the end of the puerperium
E. For the entirety of the puerperium

A

A. At least 10 days post-partum, but not during the pregnancy

This woman has 2 minor risk factors for VTE, therefore it is not recommended to anticoagulate her during her pregnancy. However the puerperium carries the highest risk of VTE of any time during pregnancy, especially just after birth. Therefore this woman will need Clexane for at least the first 10 days after birth.

100
Q

A 41 week pregnant woman attends hospital and is admitted to the labour ward for an induction of labour. The midwife performs a membrane sweep in an attempt to trigger the start of labour. 2 hours later the woman’s cervix is firm and in the middle position, is about 50% effaced, is 1cm dilated, and the baby’s head is at station -2cm. Her Bishop score is calculated to be 4.

What is the most appropriate next step in managing her labour?

A. Start an I.V. infusion of Syntocinon
B. Give a vaginal prostaglandin
C. Perform an amniotomy (artificial rupture of membranes)
D. Give I.M. Ergometrine
E. Review her in an hour
A

B. Give a vaginal prostaglandin

This woman’s Bishop score indicates labour is unlikely to start spontaneously, and that the cervix is not ‘ripe’. Vaginal prostaglandin given as either a tablet or a gel is the recommended way to induce labour after trying a membrane sweep. Amniotomy with an oxytocin infusion (Syntocinon) can be used once the cervix is more favourable, but is not the standard method recommended for induction. Ergometrine is not used until the fetus has been delivered because it causes the arteries supplying the uterus to constrict to limit post-partum haemorrhage - if this were done before the baby were born it would restrict their blood supply.

101
Q

A primiparous 28 week pregnant woman presents to the Maternal Assessment Unit with suspected contractions. The contractions are painful and coming regularly at a rate of 3 in 10 minutes. On examination the cervix is still closed and there is no evidence of rupture of membranes. The pregnancy has so far been uncomplicated.

What is the most appropriate management?

A. Perform a TVUSS and a fetal fibronectin test
B. Give erythromycin and dexamethasone
C. Give nifedipine, dexamethasone, and magnesium
D. Attempt a rescue cerclage
E. Admit her to the labour ward and provide analgesia

A

C. Give nifedipine, dexamethasone, and magnesium

In cases of preterm labour, the goal of tocolytics is not generally to stop labour totally, but to delay it enough to allow corticosteroids to take effect and mature the baby’s lungs.

102
Q

A 32 year old woman is due to undergo partial bowel resection in 3 months time. She is currently taking the COCP.

Which of the following should she do?

A. Stop taking the COCP on the day of the surgery, then don’t resume till she is mobile again afterwards
B. Switch to the POP now, then switch back 2 weeks after surgery if she prefers
C. Stop taking it 4 weeks before the surgery, then resume 2 weeks after
D. Stop taking the COCP now and practice barrier contraception until after the operation, then resume use
E. There is no need to modify her contraceptive use

A

C. Stop taking it 4 weeks before the surgery, then resume 2 weeks after

The COCP should be discontinued 4 weeks before major or immobilising surgery, and resumed 2 weeks after. In the meantime the mini POP or barrier contraception should be used.

103
Q

A 32 year old woman and her husband visit a fertility specialist because they are having trouble getting pregnant. They have been trying for 14 months every few days with no success. The woman’s cycles are 28 days long and very regular, and neither of them have any obvious causative lifestyle factors.

Which of the following should be done as standard tests?

A. Semen analysis, mid-luteal progesterone, AMH, hysterosalpingiography
B. Semen analysis, FSH, LH, oestradiol
C. Screening for HIV, Hep B and C, and chlamydia. Day 2 LH and FSH
D. Semen analysis, prolactin, TFTs, day 21 progesterone
E. Semen analysis, prolactin, AMH, and oestradiol

A

A. Semen analysis, mid-luteal progesterone, AMH, hysterosalpingiography

104
Q

A 26 year old woman presents to A&E with 8/10 rapid onset right iliac fossa pain. She is afebrile with a tender abdomen and a palpable mass in the right iliac fossa. She complains of nausea and vomits while being examined. An USS of the right iliac fossa shows the whirlpool sign.

What is the most likely diagnosis?

A. Intussusception
B. Appendicitis
C. Ovarian torsion
D. Ovarian cyst rupture
E. Volvulus
A

C. Ovarian torsion

Ovarian torsion occurs when the ovary twists around the suspensory ligament and occludes its blood supply. This presents similarly to appendicitis, though perhaps more rapidly, but the ‘whirlpool sign’ on USS differentiates it as ovarian torsion.

105
Q

A 35 year old, 30 week pregnant woman presents to A&E with a 1 day history of gradual onset, 7/10, constant, hypogastric abdominal pain. She is nauseous and has vomited several times. On examination her uterus is soft and non contractile but tender, and a speculum exam shows a closed, non-tender cervix and no evidence of ruptured membranes. Her observations are normal.

What is the most likely diagnosis?

A. Ovarian torsion
B. Appendicitis
C. Ovarian cyst rupture
D. Red degeneration of fibroids
E. Threatened miscarriage
A

D. Red degeneration of fibroids

Fibroids respond to hormones and so in pregnancy they sometimes enlarge (around 1/3 of fibroids enlarge in pregnancy). Red degeneration is where a fibroid enlarges to the point it occludes or outgrows its blood supply. This causes severe abdominal pain, nausea, and vomiting.

Management is conservative and supportive.

106
Q

Match each type of ovarian mass to the corresponding description:

A. Corpus luteum cyst
B. Endometrioma
C. Follicular cyst
D. Mature teratoma
E. Theca lutein cyst
  1. The most common ovarian cyst in young women
  2. The most common benign ovarian tumour in young women, features Rokitansky’s nodule
  3. Also known as ‘chocolate cysts’
  4. Seen during pregnancy as they grow in response to beta-hCG
  5. Particularly associated with haemorrhage
A
  1. The most common ovarian cyst in young women - C. Follicular cyst
  2. The most common benign ovarian tumour in young women, features Rokitansky’s nodule - D. Mature teratoma
  3. Also known as ‘chocolate cysts’ - B. Endometrioma
  4. Seen during pregnancy as they grow in response to beta-hCG - E. Theca lutein cyst
  5. Particularly associated with haemorrhage - A. Corpus luteum cyst
107
Q

Which of the following statements on endometrial hyperplasia is incorrect?

A. The Mirena coil is the first-line treatment for typical hyperplasia
B. The first-line treatment for atypical hyperplasia is total hysterectomy
C. It can be diagnosed on an USS
D. It may be caused by breast cancer treatment
E. Most cases of typical hyperplasia will spontaneously remit, and have <5% chance of becoming cancerous over 20 years

A

C. It can be diagnosed on an USS

Endometrial hyperplasia requires a biopsy for diagnosis. It is key to differentiate typical from atypical endometrial hyperplasia: typical hyperplasia would probably self-resolve and is very conservatively managed, whereas atypical hyperplasia requires a full hysterectomy and bilateral salpingo-oopherectomy (the latter depends on age). This is because atypical hyperplasia has a significant risk of progressing to malignancy.

‘D’ is true because tamoxifen use is associated with endometrial hyperplasia.

108
Q

A 30 year old woman presents to her GP with episodes of small volume, painless vaginal bleeding after sex. She has otherwise been entirely well. An abdominal and vaginal exam reveals no abnormalities, though on speculum the cervix looks unusual - there is a red area around the external os. She has no PMHx, was fully vaccinated as a child, and her only medication is the COCP.

What is the most likely diagnosis?

A. Sexually transmitted infection
B. Cervical intraepithelial neoplasia
C. Endometriosis
D. Cervical ectropion
E. Trauma
A

D. Cervical ectropion

Cervical ectropion is a common and benign condition (particularly associated with COCP use and pregnancy) where columnar epithelium develops on the vaginal section of the cervix. This epithelium is more delicate than the usual stratified squamous epithelium, and so can be damaged during sex leading to post-coital bleeding. This may also cause an increase in clear vaginal discharge, as columnar epithelium secretes mucous.

Cervical cancer is an important differential to rule out in this woman, especially as she is in the peak age category. However as she is fully vaccinated cervical cancer is unlikely, because this would have included an HPV vaccine.

109
Q

Which of the following correctly describes the treatment of diabetes in pregnancy?

A. Metformin should be substituted for any other diabetic drugs
B. Sulphonylureas are first line diabetes treatment in pregnancy
C. Insulin is the only medication known to be safe in pregnancy
D. Insulin should be used as monotherapy
E. Metformin may be used +/- insulin

A

E. Metformin may be used +/- insulin

110
Q

A 14 year old presents to her GP practice with her mother regarding her painful periods. They have always been very painful since beginning a year ago, and have been getting in the way of day to day life for the child. Her cycle is regular and 28 days long, and her bleeding is not especially heavy. She reports being otherwise well. The GP asks permission to perform a routine vaginal exam but the daughter refuses and is extremely uncomfortable with the idea.

How should this child be managed?

A. Trial high frequency TENS
B. Insert the Mirena coil
C. Prescribe the COCP
D. Prescribe mefanemic acid
E. Recommend a trial of OTC analgesia
A

D. Prescribe mefanemic acid

The first line treatment for dysmenorrhea is NSAIDs, and mefanemic acid is well know for being effective in reducing period pain. The Mirena coil may well be effective and can be used for dysmenorrhea as an alternative first line option, but would be an issue in this patient given her reluctance to even have a vaginal exam.

Similarly the COCP can be used as an alternative first line treatment, but given this child doesn’t need contraception, it is better to give an NSAID as there are fewer side-effects. A TENS machine may be tried in some cases but is not a standard treatment. Lastly, telling a patient to use OTC analgesia in cases where the pain has been present for some time isn’t a good option as they have probably already tried it.

111
Q

A 19 year old woman visits the GP to have the Mirena coil fitted, and asks how long she has to use condoms for before the coil starts to work?

A. 2 days
B. 4 days
C. 5 days
D. 6 days
E. 7 days
A

E. 7 days

The copper IUD is immediately effective, the POP mini-pill is effective after 48 hours, but generally speaking other hormonal contraception will not provide protection till 7 days after initiation (including COCP, Mirena, depot, and the implant).

112
Q

A 51 year old presents to her GP practice complaining of hot flushes. They began a few months ago and she is reluctant to take any hormonal treatments because of the risk of cancer.

What alternative treatment would be most appropriate?

A. Primrose oil
B. CBT
C. Topical oestrogen
D. Fluoxetine
E. Amitriptyline
A

D. Fluoxetine

A trial of an SSRI can be used to help with vasomotor symptoms in women who don’t want to use HRT or can’t.

113
Q

A 9 week pregnant woman attends her booking appointment, and as part of the consultation provides a urine sample. Culturing of the sample reveals bacteriuria, though the woman denies any urinary symptoms.

How should this woman be managed?

A. Repeat the urine dip at her next appointment
B. Reassure her that if she is asymptomatic, this is not significant
C. Prescribe Nitrofurantoin B.D.S. for 7 days
D. Prescribe Trimethoprim B.D.S. for 3 days
E. Advise hydration and hygiene measures

A

C. Prescribe Nitrofurantoin B.D.S. for 7 days

Even if it is asymptomatic, bacteriuria in pregnancy is a risk factor for pyelonephritis, stillbirth, and IUGR, and so should be treated. Trimethoprim is contraindicated in the first trimester because it is an anti-folate drug which would cause birth defects.

114
Q

Which of the following statements on HIV in pregnancy is false?

A. Ideally, HIV + women should not breastfeed
B. Intrapartum infusions of Zidovudine are recommended for all HIV + women
C. Post-exposure prophylaxis is advised for all children of HIV + mothers regardless of viral load
D. Immediate C-section is recommended for a patient with viral load >400 and SROM
E. HIV infection is not necessarily a contraindication to vaginal birth

A

B. Intrapartum infusions of Zidovudine are recommended for all HIV positive women

Intrapartum infusions of Zidovudine may be recommended depending on the woman’s viral load, but are unnecessary if the load is <50, and may not be necessary even if it is a little over.

‘A’ is true - the British HIV Association recommend that breastfeeding should be avoided, though the risk in a mother with well-controlled HIV is very small. ‘C’ is also correct - all babies born to HIV+ mothers should receive at least 2 weeks of Zidovudine therapy even if the mother’s viral load is undetectable.

‘D’ is correct - if a mother has rupture of the membranes with a viral load >400, the risk of transmission to the fetus is too high and she should have an immediate C-section. ‘E’ is also true because in a woman with viral load <50 there is no contraindication to vaginal birth, and if it is between 50-399 then it’s a grey area.

BHIVA guidelines:
https://www.bhiva.org/file/5bfd30be95deb/BHIVA-guidelines-for-the-management-of-HIV-in-pregnancy.pdf

115
Q

Which of the following supplement regimes should every pregnant women be given as a minimum?

A. 400 mcg Folate, Iron
B. 5 mg Folate, Iron
C. 400 mcg Folate, Vitamin D
D. 5 mg Folate, Iron, Magnesium
E. 400 mcg Folate, Vitamin D, Iron
A

C. 400 mcg Folate, Vitamin D

116
Q

A 10 week pregnant woman presents to hospital with severe and prolonged vomiting. She has been vomiting all day and has been unable to keep any food down. She was seen in clinic for nausea and vomiting and was given cyclizine, which has clearly not worked. On examination she is clinically dehydrated with tachycardia and slight hypotension. She is weighed and found to have lost significant weight. A VBG shows hypokalaemia and hyponatraemia. She is admitted for treatment and parenteral nutrition.

Which vitamin is it most important to replace in this woman before giving parenteral nutrition?

A. Thiamine
B. Folate
C. Iron
D. Vitamin C
E. Vitamin D
A

A. Thiamine

Women with nausea and vomiting of pregnancy (N&VP) or hyperemesis gravidarum are at increased risk of developing Wernicke’s encephalopathy. Not only do they have an increased demand by virtue of being pregnant, but if the vomiting becomes severe they become malnourished and deficient in thiamine. Administration of I.V. dextrose and parenteral nutrition is part of the treatment for severe N&VP, but will also precipitate Wernicke’s encephalopathy in a woman who is thiamine deficient. Hence thiamine must be supplemented before starting I.V. dextrose or nutrition, and must be given with all infusions of dextrose or nutrition thereafter.

117
Q

A 9 week pregnant g2p1 woman attends her local GP practice for her booking appointment. She mentions her previous pregnancy was complicated by gestational diabetes, though it remitted post-partum.

How should this woman be investigated?

A. Organise for her to have a continuous blood sugar monitoring device
B. Perform an OGTT at 24-28 weeks
C. Start metformin therapy, then assess response with an OGTT at 24-28 weeks
D. Perform an OGTT as soon as is feasible
E. Perform a urine dipstick test in the appointment

A

D. Perform an OGTT as soon as is feasible

118
Q

Decide the best next step in each of the following cases

A. Perform a smear in 3 years
B. Refer for urgent colposcopy
C. Refer for colposcopy
D. Perform a smear in 6 months
E. Perform a LLETZ
F. Perform a biopsy
G. Perform HPV testing
  1. A smear shows moderate dyskaryosis
  2. A smear shows borderline dyskaryosis
  3. A smear shows HPV negative mild dyskaryosis
  4. A 6 month pregnant woman is due for a smear
  5. 3 samples in a row come back inadequate
A
  1. A smear shows moderate dyskaryosis - B. Refer for urgent colposcopy
  2. A smear shows borderline dyskaryosis - G. Perform HPV testing
  3. A smear shows HPV negative mild dyskaryosis - A. Perform a smear in 3 years
  4. A 6 month pregnant woman is due for a smear - D. Perform a smear in 6 months
  5. 3 samples in a row come back inadequate - C. Refer for colposcopy
119
Q

A 30 year old woman presents to a sexual health clinic with suprapubic pain and irregular inter-menstrual bleeding. Examination shows abdominal tenderness, cervical excitation, and adenexal tenderness. A pregnancy test is performed and is negative.

What is the most likely diagnosis?

A. Ectopic pregnancy
B. Molar pregnancy
C. Urinary tract infection
D. Pelvic inflammatory disease
E. Pyelonephritis
A

D. Pelvic inflammatory disease

Cervical excitation is also known as cervical motion tenderness and is characterised by intense pain on palpation of the cervix. Classically there are two main things that cause cervical excitation: pelvic inflammatory disease (PID), and ectopic pregnancy. The negative pregnancy test excludes an ectopic, making this PID. In reality there are other things that may cause cervical excitation, but PID and ectopic pregnancy are the main differentials.

You would also expect to see fever, perhaps tachycardia, and discharge on a vaginal exam (possibly with a history of unsafe success) but these have been omitted from this question to make cervical excitation the focus.

120
Q

A 35 year old woman and her husband are referred to a fertility specialist having failed to conceive for 1 year. The specialist asks about the woman’s menstrual cycles, which she says are reliably 37 days long. The specialist wants to assess whether the patient is ovulating.

How should ovulation be assessed in this patient?

A. Measure day 14 progesterone
B. Measure day 21 progesterone
C. Measure day 30 progesterone
D. Plotting serial temperature measurements
E. Assessment of cervical mucous
A

C. Measure day 30 progesterone

Though the length of menstrual cycles varies, the luteal phase is kept relatively constant, therefore if a patient has regular cycles they will reliably ovulate 7 days before the end of their cycle - in this case day 30.

121
Q

A g1p0 39 year old woman who has just given birth begins to seize shortly afterwards. Her pregnancy has been complicated by pre-eclampsia and she had recently been complaining of headaches and visual disturbances.

Which medication should this patient be given first?

A. Buccal Midazolam
B. I.V. Midazolam
C. I.V. Magnesium sulphate
D. Rectal DIazepam
E. Aspirin
A

C. I.V. Magnesium sulphate

Seizures in eclampsia are both prevented and managed with Magnesium sulphate, instead of with I.V. Midazolam as in normal seizures.

122
Q

A 36 year old woman presents to a fertility specialist regarding inability to conceive. An USS shows several large fibroids which the specialist thinks may be impairing her fertility.

How should the fibroids be definitively treated?

A. A GnRH agonist
B. Endometrial ablation
C. Hysterectomy
D. Trans-cervical resection of endometrium
E. Myomectomy
A

E. Myomectomy

Myomectomy is the only option for fibroid treatment that will preserve fertility. Other options will either damage/ remove the endometrium, or shutdown the HPG axis making the woman infertile. Uterine artery embolisation is an option for fibroid treatment but its effect on pregnancy is unclear so myomectomy is used.

123
Q

Which of the following is an indication for using cyclical rather than continuous HRT?

A. If a women still has her womb
B. If a woman is >50 years old
C. If a woman is <50 years old
D. If a woman is still having periods
E. Women with pronounced menopausal symptoms
A

D. If a woman is still having periods

The purpose of cyclical HRT is to still have periods, allowing the woman to keep track of her progression through the menopause. Periods are triggered not by withdrawal of a medication, but by addition: oestrogen is taken continuously and a progestogen is added for 14 days. The progestogen triggers a period and can be added either once or month or once every 3 months (the latter is usually for women having irregular periods).

NB: You use cyclical HRT till a woman is officially post-menopausal (12 months of amenorrhea)

124
Q

A 14 year old is brought to see the GP by her mother as she has not started having periods yet. The girl is a normal height and weight for her age having recently started a growth spurt, and there is evident appropriate breast development. The girl describes cyclical suprapubic deep pain which comes once a month and lasts for 3 days.

What is the most likely diagnosis?

A. Idiopathic dysmenorrhea
B. Imperforate hymen
C. Congenital adrenal hyperplasia
D. Endometriosis
E. Irritable bowel syndrome
A

B. Imperforate hymen

125
Q

Which of the following foods should always be avoided in pregnancy?

A. Mayonnaise
B. Cream cheese
C. Shellfish
D. Liver
E. Coffee
A

D. Liver

Cream cheese is a red herring, it is pasteurised so it is safe. Most foods that are advised against in pregnancy carry an infection risk, and so are safe if thoroughly cooked or pasteurised. However liver is dangerous because it is so high in Vitamin A which is a teratogenic.

126
Q

A 37 week pregnant woman presents to the labour ward for induction of labour. Her labour does not progress quickly despite augmentation with Syntocinon. She is pregnant with twins and her pregnancy has so far been complicated by polyhydramnios and fibroids.

Which of the following complications is this women most at risk of?

A. Eclampsia
B. Puerperal psychosis
C. Chorioamnionitis
D. Shoulder dystocia
E. Post-partum haemorrhage
A

E. Post-partum haemorrhage

All the mentioned complications are risk factors for post-partum haemorrhage - defined as more than 500ml loss of blood from the genital tract within 24 hours of the birth of a baby.

127
Q

An amniotomy is performed on a 40 week pregnant woman on the labour ward in an attempt to induce labour, as the patient has a contraindication to vaginal prostaglandin. Shortly afterwards the CTG trace becomes pathological, and on examination the midwife can see the umbilical cord protruding. through the cervical os. The midwife presses the emergency button.

How should this situation be managed?

A. Place her in McRoberts position and apply suprapubic pressure
B. Elevate the presenting part towards the uterus, get the patient on all fours, take her to theatre for emergency C-section
C. First attempt to return the cord to inside of the uterus, if this fails take her to theatre for emergency C-section
D. Take the patient to theatre immediately for C-section
E. First attempt to return the cord to inside of the uterus, if this fails then place the patient on all fours and elevate the presenting fetal part

A

B. Elevate the presenting part towards the uterus, get the patient on all fours, take her to theatre for emergency C-section

A cord prolapse is an obstetric emergency, as compression of the cord can cause fetal hypoxia and death. The most important initial intervention is to relieve pressure from the baby on the cord by pushing the presenting part back towards the uterus, and to put the woman on all fours. She should then be taken to theatre for C-section in most cases, though instrumental delivery may sometimes be appropriate (but not here as it is too early). Tocolytics (e.g. Terbutaline) are also be given.

NB: It is important to not try and push the cord back into the uterus as this may result in further compression.

NB: The amniotomy (assisted rupture of membranes) is significant here because it is associated with cord prolapse

128
Q

A 25 year old 8 week pregnant woman presents to the maternal assessment unit with abdominal cramping and vaginal bleeding. A cervical exam reveals an open cervical os with some bloody discharge. A TVUSS reveals a normal crown-rump length for this gestational age (17mm) and no cardiac activity.

How should this patient be managed?

A. Counsel her that she is miscarrying, tell her to take a pregnancy test in 3 weeks and return if it is still positive
B. Counsel her that she is miscarrying and discuss expectant management
C. Give oral or vaginal Misoprostol
D. Re-scan the woman again in 1 week
E. Refer her for an evacuation under local anaesthesia

A

B. Counsel her that she is miscarrying and discuss expectant management

This is a history of an inevitable miscarriage, as the cervical os is open and there is no cardiac activity. The first line management for miscarriage is expectant which may last 7-14 days. However the woman’s choice is very important, and medical management in hospital involving vaginal Misoprostol to pass the products of conception may be used instead.

129
Q

A g3p2 35 year old woman attends her booking appointment at 9 weeks. She mentions to the midwife that her last baby was very ill soon after being born, and the doctors told her it was from a bacteria that had colonised her vagina. The midwife checks her notes and sees it was a Group B Streptococcus (GBS) infection.

How should this woman be managed?

A. Offer intra-partum Benzylpenicillin
B. Give Benzylpenicillin prophylaxis from 20 weeks till term, and then intra-partum
C. Offer maternal GBS testing at 20 weeks, and intra-partum Benzylpenicillin if positive
D. Offer maternal GBS testing at 35 weeks, and intra-partum Benzylpenicillin if positive
E. Treat her with a 10 day course of Erythromycin, and offer intra-partum Benzylpenicillin

A

A. Offer intra-partum antibiotics

If a mother has had a child with early onset (within 24 hours of birth) GBS infection before, then she should be offered intra-partum antibiotics (IPA) in subsequent pregnancies. This is done regardless of the results of GBS testing.

If a woman has GBS bacteriuria then she should be offered IPA. If a woman has a UTI with GBS, she should be treated at the time of diagnosis, and also offered IPA.

130
Q

A g2p1 29 year old woman attends her booking appointment with the midwife, who sees in her notes that asymptomatic Group B Streptococcus (GBS) colonisation was found incidentally in a previous pregnancy, though the baby was not infected.

How should this woman be managed?

A. Offer GBS swabs along with each routine urine dipstick at antenatal appointments
B. Offer a GBS swab at the booking appointment
C. Swab the baby post-natally and give Benzyl penicillin if swabs are positive
D. Offer a GBS swab at 35 weeks
E. Offer intra-partum Benzylpenicillin

A

D. Offer a GBS swab at 35 weeks

This woman has a ~50% chance of GBS colonisation in a subsequent pregnancy, and so should be offered a swab for GBS. Whilst a woman may choose intra-partum antibiotics (IAP) without testing, a swab should at least be offered. Swabs are done between 35-37 weeks, or 3-5 weeks before expected delivery if the baby is expected earlier.

It is worth noting that a positive swab is not a very useful result, as it indicates a chance of 1 in 400 of early-onset GBS disease of the newborn. However a negative swab makes this risk 1 in 5000 so it is a useful test for ruling out the need for intra-partum antibiotics, but not so much for ruling them in.

If a woman has GBS bacteriuria then she should be offered IPA. If a woman has a UTI with GBS, she should be treated at the time of diagnosis, and also offered IPA.

131
Q

A 30 week pregnant g3p2 woman visits the doctor for a routine antenatal appointment. The abdomen is palpated and her baby is found to be in breech. The doctor informs her that this is not necessarily an issue yet, and she will be offered ECV at a later date if the baby is still in breech.

When would this woman be offered ECV?

A. 28 weeks
B. 30 weeks 
C. 33 weeks
D. 36 weeks
E. 37 weeks
A

E. 37 weeks

As she is a multip, she is offered ECV at 37 weeks, as opposed to a primip who would be offered it at 36 weeks. This is because multips have a higher chance of their baby spontaneously moving out of breech, so ECV is delayed by one week for them.

132
Q

Match each contraceptive method with its primary mode of action (options may be used twice):

A. Inhibits fertilisation by impairing sperm
B. Inhibits implantation
C. Inhibits ovulation
D. Thickens cervical mucous and thins the endometrium

  1. COCP
  2. IUS
  3. IUD as emergency contraception
  4. IUD as regular contraception
  5. The progesterone implant
A
  1. COCP - C. Inhibits ovulation
  2. IUS - D. Thickens cervical mucous and thins the endometrium
  3. IUD as emergency contraception - B. Inhibits implantation
  4. IUD as regular contraception - A. Inhibits fertilisation by impairing sperm
  5. The progesterone implant - C. Inhibits ovulation

NB: The POP mini-pill was excluded here because its effects vary with preparation and are more complex

133
Q

A 25 week pregnant woman presents with small painless ante-partum haemorrhage. She is haemodynamically stable, and has already had bloods taken including FBC, G&S and a Kleihauer test.

Which next investigation is most important in this woman?

A. A vaginal examination
B. An abdominal USS
C. A coagulation screen
D. An abdominal exam
E. TVUSS
A

E. TVUSS

In ante-partum haemorrhage it is most important to ensure the health of mother and baby first. After that it is important to establish the placental location, which is done using TVUSS.

A CTG is not listed here as an option but is also an important test, arguably more important as fetal well-being should be established as a priority.

134
Q

A woman presents to her GP with vaginal bleeding. She gave birth 3 weeks ago and is worried that she is still having to wear sanitary pads constantly due to bloody discharge. Abdominal palpation is normal, and vaginal examination reveals some bloody discharge.

How should this woman be managed?

A. Perform a vaginal swab
B. Perform a urine dipstick
C. Reassure her this is normal
D. Admit her and give I.V. Ceftriaxone 
E. Perform a TVUSS
A

C. Reassure her this is normal

Lochia refers to the bloody discharge after pregnancy and normally lasts 6-8 weeks, though generally should not be blood stained after 4 weeks. There are no signs of infection or retained products in this woman, so this is not worrying and she should be reassured that this is normal.

135
Q

A woman who gave birth 8 weeks ago visits her GP to ask about contraception. She began having sex with her husband again 4 weeks after giving birth without protection, and wants to know if she can now use the COCP. She is currently using a combination of breastfeeding and formula milk feeds.

How should this patient be managed?

A. Advise that she takes a pregnancy test, prescribe the COCP
B. No pregnancy test is needed, prescribe the COCP
C. Advise that she takes a pregnancy test, discuss other methods of contraception as COCP is contraindicated
D. No pregnancy test is needed, counsel her that intrauterine contraception is recommended for the first 6 months post-partum
E. Advise that she takes a pregnancy test

A

A. Advise that she takes a pregnancy test, prescribe the COCP

A woman is naturally protected against pregnancy for the first 3 weeks post-partum, but not afterwards. This woman will therefore need a pregnancy test.

Whilst the COCP is absolutely contraindicated (UKMEC 4) for breastfeeding mothers within the puerperium, it is generally accepted that the benefits outweigh the risks (UKMEC 2) after the first 6 weeks post-partum.

NB: This question is modified from a PassMed question

136
Q

A woman visits her GP with a complaint of vaginal discharge. She underwent a C-section 1 week ago and since then has been experiencing bloody discharge from her vagina. The GP performs an abdominal examination, and feels a soft uterus around the level of the umbilicus. A vaginal and pelvic exam reveals offensive discharge and uterine tenderness. Observations show the mother is tachycardic and has a fever, but are otherwise normal.

What is the most likely diagnosis?

A. Surgical site infection
B. Retained products of conception
C. Pelvic inflammatory disease
D. Urinary tract infection
E. Pyelonephritis
A

B. Retained products of conception

A soft or ‘boggy’ uterus shortly after birth indicates uterine atony or retained products of conception. This may cause either primary (within 24 hours) or secondary (24 hours - 12 weeks) post-partum haemorrhage. In this case there are clearly signs of infection which points to retained products of conception that have now become infected causing endometritis. This woman would require examination + ERPC under anaesthesia.

137
Q

Which of the following tests are routinely performed at a booking appointment?

A. BMI, blood pressure, urine dip, blood test for infections, blood test for thalassaemia
B. Dating ultrasound scan, blood test for infections
C. Ultrasound scan for major abnormalities,
D. BMI, blood pressure, urine dip, FBC, LFTs, U&Es
E. Blood test for infections, discuss family and personal medical history, auscultate for fetal heartbeat

A

A. BMI, blood pressure, urine dip, blood test for infections, blood test for thalassaemia

138
Q

A 40 year old woman presents with increasingly heavy and painful periods. She has a PMHx of endometriosis which was treated with laparoscopic destruction of the lesions and has not troubled her since. On palpation, her uterus is mildly enlarged and tender, though has no palpable masses within.

Given the most likely cause, how can this condition be definitively diagnosed?

A. Using an USS
B. On a speculum exam
C. On clinical symptoms
D. Using urinanalysis
E. Using an MRI
A

E. MRI

This is a history of adenomyosis - essentially endometriosis ocurring within the myometrium. It causes uterine tenderness and enlargement, as well as heavy and painful periods. USS can be helpful, but this condition can only be definitely diagnosed on MRI. There is a variety of treatments including: the IUS, mefanemic acid and tranexamic acid to control the pain and bleeding, and hysterectomy as the most comprehensive management.

139
Q

A 38 year old woman with chronic hypertension visits her GP to ask how the management of her condition would change if she got pregnant, as she is planning to start a family with her husband. She is currently taking Ramipril to control her blood pressure.

How should this woman’s hypertension be managed in pregnancy?

A. Swap from Ramipril to an Angiotensin-receptor blocker
B. Continue with Rampiril and give aspirin from 12 weeks gestation
C. Continue with Rampiril as the risks of disturbing her hypertension management outweigh the rewards
D. Switch Ramipril for Nifedipine, or Methyldopa if Nifedipine is unsuitable
E. Switch Ramipril for Labetalol and give aspirin from 12 weeks gestation

A

E. Switch Ramipril for Labetalol and give aspirin from 12 weeks gestation

Ramipril is an ACE inhibitor, a class of drugs which is teratogenic in pregnancy. Accordingly it should be swapped with Labetalol which is the first line anti-hypertensive in pregnancy. Nifedipine is given if Labetalol is unsuitable, and Methyldopa is the third-line treatment.

Chronic hypertension is a high risk factor for preeclampsia development, so 75-150 mg aspirin is given from 12 weeks.

140
Q

A 26 year old, previously well woman presents to her GP with an 8 week history of amenorrhea. A urinary pregnancy test is positive, but a TVUSS cannot locate an intrauterine pregnancy. Same day assessment in hospital reveals a left-sided tubal mass of 40mm with a detectable heartbeat, and an ectopic pregnancy is diagnosed. The serum beta hCG level is 1900.

How should this woman be managed?

A. Repeat the serum b-hCG measurement in 48 hours
B. Salpingectomy
C. Salpingotomy
D. Offer a choice of expectant management or a single I.M. dose of 5mg/ square metre methotrexate
E. Offer a choice of laparoscopy or a single I.M. dose of 5mg/ square metre methotrexate

A

B. Salpingectomy

Repeating the b-hCG measurement may be useful when considering expectant management, but that is not an option here because the tubal pregnancy is >35mm and has a heartbeat.

The size of the mass and presence of a heartbeat also excluded medical management. A salpingectomy is usually preferred to ensure none of the ectopic pregnancy is left behind, and would usually be performed unless the other Fallopian tube is impaired.

Beta hCG may be used to guide treatment:
<1000 IU/L: expectant management
<1500 IU/L: patient’s choice between methotrextate and expectant management
1500-5000 IU/L: patient’s choice between methotrexate and surgery
>5000 IU/L: surgery
NB: The ectopic pregnancy must not be larger than 35mm, there must be no visible heartbeat, and no pain or haemodynamic instability, otherwise surgery is indicated

141
Q

A 14 week pregnant woman is noted to have a blood pressure of 153/96 mmHg at an antenatal appointment. She is given an ambulatory monitoring device which confirms the reading.

What is the most likely diagnosis?

A. Pre-eclampsia
B. Gestational hypertension
C. Chronic hypertension
D. White-coat hypertension
E. Renal disease
A

C. Chronic hypertension

The reason this is chronic hypertension and not gestational hypertension, is because gestational hypertension is defined as now-onset hypertension after 20 weeks. By definition, this cannot be gestational hypertension and must have been long-standing. There is nothing in the history to suggest renal disease, or proteinuria indicating preeclampsia.

NB: This is modified from a PassMed question

142
Q

An 18 year old woman taking the desogestrel POP ‘Cerazette’ visits her GP worried because she has missed a pill. She was supposed to take the pill at 09:00 and it is now 14:00. She had unprotected sex with her boyfriend in the interim before realising she had not taken the pill.

What should the GP do?

A. Tell her to take the pill now, and reassure her no further action is needed
B. Discuss insertion of the IUD as both emergency and long-term contraception
C. Offer Levonorgesterel emergency contraception, then tell her to take the POP now, then the next one at the normal time and use barrier contraceptive till there is 48 hours of POP cover
D. Reassure her that missing one pill is not significant, and tell her to take her contraception as normal
E. Tell her to take the pill now, then the next one at the normal time and use barrier contraceptive till there is 48 hours of POP cover, then offer a pregnancy test

A

A. Tell her to take the pill now, and reassure her no further action is needed

Desogestrel POPs such as Cerazette can be taken within a 12 hour window, which this girl has not actually missed.

If she had missed a POP, then she would need to take the missed pill as soon as she remembered, then take the rest of the pills normally (even if that means having 2 in 1 day). Barrier contraception should be used until 48 hours after re-starting the pill, and emergency contraception should be offered if there is unprotected sex before that time.

143
Q

A 15 week pregnant woman visits the maternal assessment unit worried that she has come into contact with a child with chickenpox. She did not have the infection as a child and has not been immunised. The child she had contact with was asymptomatic at the time, but developed a rash two days later.

How should this woman be managed?

A. Measure IgM and give VZIG if it is raised
B. Give Varicella-Zoster immunoglobulin
C. Reassure her that the child would not have been infectious pre-rash and so would not have transmitted it to her
D. Give oral aciclovir
E. Admit her and give intravenous aciclovir

A

B. Give Varicella-Zoster immunoglobulin

Children infected with VZV are most infectious before the typical chickenpox rash develops. Accordingly this woman needs immunoglobulin prophylaxis because there is a risk of congenital Varicella syndrome (though it is very small). Measuring IgM would be unhelpful because that would only be positive once an infection has developed, and the goal is to prevent that from occurring. Furthermore she says she did not have the infection as a child, though if there were any uncertainty this could be checked by measuring VZV IgG.

Oral aciclovir is not generally indicated unless the woman develops the chickenpox rash, though this is contentious as some hospitals will give prophylactic aciclovir to exposed women.

I.V. aciclovir is reserved for severe ZVZ infection, e.g. pneumonitis which is a significant complication in adults.

144
Q

When do periods resume after birth?

A

It depends on whether the mother breastfeeds, as high prolactin causes amenorrhea. If the mother doesn’t breastfeed they may return as early as 5-6 weeks post-partum.

145
Q

An 10 week pregnant woman attends her booking appointment, and routine bloods find a microcytic anaemia of 105 g/L.

How should this woman be managed?

A. Trial oral iron tablets and reassess her in 2 weeks
B. Perform iron studies
C. This is a normal finding in pregnancy
D. Check B12 and Folate
E. Refer her for an OGD
A

A. Trial oral iron tablets and reassess her in 2 weeks

According to RCOG, a normocytic or microcytic anaemia in pregnancy is managed with a 2 week trial of oral iron supplementation, before any investigations are done.

This is not a normal finding in pregnancy, there may be an anaemia but the haemoglobin shouldn’t be below 110 in the first trimester, 105 in the second, and 100 in the third.

146
Q

What are the three criteria for PCOS?

A

Ovulatory dysfunction
Clinical or biochemical evidence of hyperandrogenism
Polycystic ovaries

Two of these 3 criteria must be fulfilled for a diagnosis; these are known as the Rotterdam criteria

147
Q

A woman who is 1 week post-partum presents to A&E with offensive vaginal discharge that includes clots. She is febrile and tachycardic, and the doctor suspects endometritis.

Which of the following is the most significant risk factor for endometritis?

A. Intrapartum antibiotic use
B. Use of instruments in delivery
C. Advanced maternal age
D. Premature rupture of membranes
E. Caesarean section delivery
A

E. Caesarean section delivery

C-section delivery greatly increases the risk of endometritis: the incidence after normal delivery is 1-3%, whereas the incidence after C-section is between 15-40%.

148
Q

The miwdwife assesses a woman post-partum for perineal tears, and finds there is a tear involving the perineal muscles and the outermost portion of the external anal sphincter.

What grade tear is this?

A. 2
B. 3a
C. 3b
D. 3c
E. 4
A

B. 3a

RCOG grades tears as follows:

First-degree tear: Injury to perineal skin and/or vaginal mucosa.
Second-degree tear: Injury to perineum involving perineal muscles but not involving the anal
sphincter.
Third-degree tear: Injury to perineum involving the anal sphincter complex:
Grade 3a tear: Less than 50% of external anal sphincter (EAS) thickness torn.
Grade 3b tear: More than 50% of EAS thickness torn.
Grade 3c tear: Both EAS and internal anal sphincter (IAS) torn.
Fourth-degree tear: Injury to perineum involving the anal sphincter complex (EAS and IAS)
and anorectal mucosa

149
Q

The miwdwife assesses a woman post-partum for perineal tears, and finds there is a tear involving the perineal muscles and anal mucosa.

What grade tear is this?

A. 2
B. 3a
C. 3b
D. 3c
E. 4
A

E. 4

RCOG grades tears as follows:

First-degree tear: Injury to perineal skin and/or vaginal mucosa.
Second-degree tear: Injury to perineum involving perineal muscles but not involving the anal
sphincter.
Third-degree tear: Injury to perineum involving the anal sphincter complex:
Grade 3a tear: Less than 50% of external anal sphincter (EAS) thickness torn.
Grade 3b tear: More than 50% of EAS thickness torn.
Grade 3c tear: Both EAS and internal anal sphincter (IAS) torn.
Fourth-degree tear: Injury to perineum involving the anal sphincter complex (EAS and IAS)
and anorectal mucosa

150
Q

Match each infectious agent with its treatment (one treatment is used twice):

A. Topical Clotrimazole
B. Oral Metronidazole
C. Doxycycline
D. Oral Azithromycin and intramuscular Ceftriaxone

  1. Trichomonas vaginalis
  2. Chlamydia trachomatis
  3. Neisseria gonorrhea
  4. Bacterial vaginosis
  5. Candida albicans
A
  1. Trichomonas vaginalis - B. Oral Metronidazole
  2. Chlamydia trachomatis - C. Doxycycline
  3. Neisseria gonorrhea - D. Oral Azithromycin and intramuscular Ceftriaxone
  4. Bacterial vaginosis - B. Oral Metronidazole
  5. Candida albicans - A. Topical Clotrimazole
151
Q

A 35 year old woman with uterine prolapse is referred to surgeons after topical oestrogen and pelvic floor training fail to control her symptoms. Her symptoms began after the traumatic birth of her first child, but she wishes to have more. She does not currently experience incontinence.

What would be the most suitable surgical management?

A. A Manchester repair
B. A hysterectomy with vaginal sacrospinous fixation
C. Anterior repair with colposuspension
D. Vaginal sacrospinous hysteropexy with sutures
E. Anterior repair

A

D. Vaginal sacrospinous hysteropexy with sutures

There are two surgical options listed by NICE for rectifying uterine prolapse that allow women to have children afterwards: vaginal sacrospinous hysteropexy with sutures, and sacro-hysteropexy with mesh.

If the woman does not wish to have more children, then a Manchester repair or hysterectomy +/- sacrospinous fixation may also be appropriate.

NICE specifically states that surgery to treat incontinence should not be performed prophylactically.

NICE guidelines on prolapse and incontinence:
https://www.nice.org.uk/guidance/ng123/chapter/Recommendations#surgical-management-of-pelvic-organ-prolapse

152
Q

Which of the following methods of contraception is the only one with a proven association with weight gain?

A. COCP
B. POP
C. Depo-Provera
D. IUS
E. Implant
A

C. Depo-Provera

Depo-Provera is the progesterone depot injection and has some significant side-effects not seen with other methods of contraception, including weight gain and osteoporosis.

153
Q

Which test should be performed in all women presenting with new-onset urinary incontinence?

A

Urine dipstick to rule out UTI which can cause temporary incontinence

154
Q

An 11 week pregnant woman presents to A&E with suprapubic abdominal cramping and some vaginal bleeding. A TVUSS shows a crown-rump length of 20mm, without a detectable heartbeat. A speculum exam shows a closed cervical os, and a missed miscarriage is diagnosed. The woman asks that the products of conception be actively removed, as she does not want to wait to pass them at home.

How should this miscarriage be managed?

A. Vaginal Mifepristone
B. Manual vacuum aspiration under local anaesthetic
C. Vaginal Misoprostol
D. Surgical evacuation of products under general
E. Oral Mifepristone

A

C. Vaginal Misoprostol

The first-line management of miscarriage according to NICE is expectant: tell them to go home and return for assessment in 7-14 days. However there are several indications for active medical management:

Previous traumatic/ adverse outcome of pregnancy (APH, miscarriage)
High risk of haemorrhage (e.g. coagulopathy, late first trimester)
High susceptibility to consequences of infection (e.g. coagulopathy, unable to receive blood transfusion)
Evidence of infection
Preference of the patient

Medical management is vaginal (or oral) Misoprostol to stimulate the passage of the products of conception.

Surgical management - if required - consists of manual vacuum aspiration under local anaesthetic, or surgical evacuation under general anaesthetic.

NB: Women with a miscarriage should be told to take a pregnancy test 3 weeks after their management has concluded to exclude molar or ectopic pregnancy

155
Q

Which of the following is a normal feature of labour?

A. A 3/5 palpable head after engagement in the pelvis
B. Transverse head position upon engagement in the pelvis
C. A brow presentation
D. A dilation from 4 to 5 cm in 4 hours
E. Presentation of the rear of the baby

A

B. Transverse head position upon engagement in the pelvis

The fetal head enters the pelvis transversely because the widest diameter of the pelvic inlet is transverse. Once engaged, the baby rotates, usually to the Occiput anterior (OA) position. It sometimes rotates to the Occiput posterior position which makes for a trickier delivery (though most spontaneously revert to OA).

A 3/5 abdominally palpable head is by definition not engaged in the pelvis. A brow presentation occurs when the fetal neck is extended instead of flexed (a vertex presentation is normal and less complicated). Primip women should dilate at a rate of ~1cm/hour once at 4cm, and multips at 2cm per hour (this will vary, but should certainly be greater than 1cm/4 hours). Lastly, the baby should present head first (cephalic) not rear first (breech).

156
Q

A 32 year old woman presents to her GP with amenorrhea. She should have had her period 4 weeks ago but has not. A urine pregnancy test is negative. She has been previously well, and her only significant PMHx is a recent surgical TOP.

What is the likely cause of her amenorrhea?

A. High Prolactin levels
B. PCOS
C. Endometritis
D. Asherman's syndrome
E. Hypothyroidism
A

D. Asherman’s syndrome

Asherman’s syndrome is the presence of intra-uterine adhesions. These adhesions can prevent the passage of blood during the menstrual cycle, and impair fertility. It is classically caused by excessive curettage at a TOP or ERPC, though may also be caused by irradiation, myomectomy, schistosomiasis, or tuberculous endometritis.

157
Q

Which of the following sets of tests should be conducted in all women with recurrent miscarriage?

A. Parental karyotyping, thrombophilia screen, STI screen
B. STI screen, pelvic USS, thrombophilia screen
C. Hysteroscopy, STI screen, thrombophilia screen
D. Antiphospholipid antibodies, pelvic USS, thrombophilia screen, cytogenetic analysis on products of conception
E. Hysterosalpingiogram, day 21 progesterone, USS of the ovaries

A

D. Antiphospholipid antibodies, pelvic USS, thrombophilia screen, cytogenetic analysis on products of conception

https://www.rcog.org.uk/globalassets/documents/guidelines/gtg_17.pdf

158
Q

Match each form of emergency contraception with the time within which it may be used:

A. Levonorgesterel
B. Ullipristal acetate
C. Copper IUD

  1. Within 120 hours of sex
  2. Within 120 hours of sex or ovulation (whichever is latest)
  3. Within 72 hours of sex
A
  1. Within 120 hours of sex - B. Ullipristal acetate
  2. Within 120 hours of sex or ovulation (whichever is latest) - C. Copper IUD
  3. Within 72 hours of sex - A. Levonorgesterel
159
Q

A 30 year old woman presents to A&E with acute onset RIF pain that came on during sex. The pain is vaguely suprapubic and 7/10 in severity. Basic observations show HR is 84 and BP is 123/87. An USS shows free fluid in the pelvis. She has no PMHx, but has a Mirena coil in situ.

What is the most likely diagnosis?

A. Appendicitis
B. Ruptured ectopic pregnancy
C. Ovarian torsion
D. Ruptured ovarian cyst
E. Meig's syndrome
A

D. Ruptured ovarian cyst

This is a tricky question, and multiple options given here would need to be ruled out. However the sudden onset pain during intercourse is classic for a rupture ovarian cyst, and the fact that this woman is generally well makes a more catastrophic diagnosis like ruptured ectopic or appendicitis less likely.

160
Q

What sign seen on USS indicates a dichorionic pregnancy?

Which sign would indicate a monochorionic diamniotic pregnancy?

A

The Lambda sign

The ‘T’ sign

161
Q

A 25 year old woman presents with a 4 month history of irregular periods, having had only 2 in that time. On examination she has some unusual facial hair and acne, and patches of dark velvety skin in her armpits. Her BMI is measured at 23.8, and, blood tests and an USS of her ovaries confirm PCOS.

How should this patient’s symptoms be managed?

A. Advise diet and exercise modifications, and give metformin
B. Prescribe the COCP
C. Advise diet and exercise to reduce weight and control symptoms
D. Prescribe letrozole
E. Prescribe clomiphene

A

B. Prescribe the COCP

The COCP is effective at regulating menstruation and the symptoms of PCOS. Diet and exercise modifications to reduce weight would not be helpful here as the woman is of a normal BMI. Letrozole, clomiphene, and metformin are helpful for fertility in patients with PCOS, but there is no indication that pregnancy is a priority for this patient.

162
Q

A 15 year old girl visits the GP because she has noticed some vaginal discharge associated with itching. On examination the discharge is cheese-like. The doctor notices that the girl appears to have had parts of her labia removed and infundibulated. The girl mentions it and asks the doctor not to tell anyone. She is very mature and would clearly be deemed Gillick competent to make medical decisions.

How should the doctor proceed?

A. Contact the police to make a report
B. Try and persuade the girl to bring her parents to the GP practice for a discussion
C. Discuss with the girl whether she consented to the procedure
D. Consult a colleague and weigh up the benefits and potential harm of disclosing information to the authorities
E. Ask the family to come in to discuss de-infundibulating the girl

A

A. Contact the police to make a report

This is a very straightforward case because doctors are legally compelled to inform the police if they become aware of female genital mutilation of a child. It is important to remember that this does not apply to adults, who may still need medical attention and support, but there is no legal duty to inform the police and social services.

163
Q

Which of the following may lead to peri-hepatitis and peri-hepatic adhesions?

A. Ectopic pregnancy
B. Pelvic inflammatory disease
C. Pyelonephritis
D. Meig's syndrome
E. HELLP syndrome
A

B. Pelvic inflammatory disease

Peri-hepatic adhesions are considered relatively specific to PID and are caused by a complication known as Fitz-Hugh-Curtis syndrome. This is an inflammation of the liver capsule arising in a case of PID which causes the formation of peri-hepatic adhesions.

164
Q

A 45 year old woman presents with urinary incontinence characterised by a sudden feeling of needing to urinate, which often doesn’t give her time to make it to the bathroom.

Which of the following would be the best first-line treatment for this incontinence?

A. Prescribe a course of pelvic floor exercises
B. Ask her to keep a bladder diary
C. A minimum 6 week course of bladder training
D. Prescribe Oxybutynin
E. Offer bladder wall botulinim injection

A

C. A minimum 6 week course of bladder training

This is a history of urge incontinence - also called overactive bladder - which is caused by detrusor muscle overactivity. The first line management of this condition, in conjunction with lifestyle modification (e.g. decreasing caffeine intake), is a course of bladder training.

165
Q

Match each description to the corresponding type of ovarian growth:

A. Most common ovarian tumour
B. May have a range of mature cell types within the growth
C. Most common growth associated with Meig’s syndrome

  1. Fibroma
  2. Serous cystadenoma
  3. Dermoid cyst
A
  1. Fibroma - C. Most common growth associated with Meig’s syndrome
  2. Serous cystadenoma - A. Most common ovarian tumour
  3. Dermoid cyst - B. May have a range of mature cell types within the growth
166
Q

A 47 year old woman presents to A&E with a 2 day history of shortness of breath that has come on gradually. On auscultation there are reduced breath sounds over the right lung base and it is stony dull to percussion. Abdominal examination also reveals some general abdominal swelling and shifting dullness. A mass can also be felt in the right iliac fossa.

What is the most likely cause of the mass felt?

A. Ovarian carcinoma
B. Serous cystadenoma
C. Fibroma
D. Endometrioma
E. Non-ruptured follicle
A

C. Fibroma

This is a description of Meig’s syndrome - a fibroma with associated ascites and pleural effusion. It is not well understood why the effusion and ascites occurs secondary to the fibroma. This may be caused by a range of tumours, but classically and most commonly by fibromas.

167
Q

An 8 week pregnant woman presents to hospital with a 4 day history of vomiting. She has been experiencing some nausea and vomiting of pregnancy and had been prescribed Promethazine but is still vomiting. She appears clinically dehydrated and a urinary test reveals ketonuria.

How should this woman be managed?

A. Trial Metoclopramide for 3 days, and ask her to return if symptoms persist
B. Admit her to hospital
C. Give her vitamin B supplementation and prescribe Ondansetron in addition to Promethazine
D. Weigh her, and admit her if there is a weight loss of >10%
E. Take a set of bloods including a blood gas, forward the results to her GP, and ask that she be managed in primary care

A

B. Admit her to hospital

This woman is presenting with an exacerbation of nausea and vomiting of pregnancy (NVP) and possible hyperemesis gravidarum. There are several criteria for admission to hospital:

NVP that persists and prevents the patient from keeping oral anti-emetics down
NVP causing weight loss >5 % or ketonuria despite oral anti-emetics
Confirmed or suspected co-morbidity

This patient will therefore need admitting to hospital. Many of the steps mentioned in the other options will be required, but the important point is that she be admitted.

168
Q

A 51 year old woman presents to her GP with hot flushes. She has not had a period in 14 months and realised a while back she was going through the menopause. She has been fairly well throughout her life and has no relevant PMHx. She discusses HRT with the GP and opts for a continuous transdermal preparation of oestrogen.

Which of the following should be added to the oestrogen?

A. The Mirena coil
B. Depo-Provera
C. The Mini-pill
D. Fluoxetine
E. No additional medication is required
A

A. The Mirena coil

It is fair to infer from the lack of PMHx that this woman has not had a hysterectomy, in which case her oestrogen preparation will need a progestogen adding to prevent endometrial proliferation and malignancy. The Mirena coil is used for this purpose, and provides a progestogen locally where it is needed.

Depo-provera would not be recommended because of the risk of osteoporosis linked to it. The Mini-pill is not recommended for the progestogen component of HRT because of an absence of data for its efficacy. Fluoxetine can be used to relieve some symptoms of the menopause but does not replace a progestogen.

NB: This is modified from a PassMed question

169
Q

Which of the following is not a risk factor for cervical cancer?

A. Smoking
B. Increased parity
C. Early start to sexual activity
D. HPV 6 and 11
E. COCP use
A

D. HPV 6 and 11

HPV subtypes 16 and 18 are the most high risk for cervical cancer; 6 and 11 are not associated with cancer but are responsible for the majority of HPV genital warts. All the other factors increase the risk of cervical cancer, though infection with HPV 16 and 18 is by far the biggest risk factor.

170
Q

The obstetric registrar is called urgently to see a woman whose CTG trace has become pathological. Her membranes ruptured a few minutes ago, and shortly afterwards she began to bleed from her vagina, though denied pain not associated with contractions. The CTG trace showed a single prolonged deceleration. Her pregnancy up until now has been uncomplicated, with normal antenatal scans.

What is the most likely cause of the CTG deceleration?

A. Cord prolapse
B. Placental abruption
C. Vasa praevia
D. Amniotic fluid embolism
E. Placenta praevia
A

C. Vasa praevia

The classic triad of vasa praevia is rupture of membranes, painless vaginal bleeding, and fetal bradycardia or death. Vasa praevia is a condition where the fetal blood vessels run over or near the internal cervical os. When the membranes rupture, these vessels rupture too and may cause rapid exsanguination and death of the fetus, though with no effect on the health of the mother.

171
Q

A 34 year old woman has an abnormal routine cervical swab showing moderate dyskaryosis. HPV testing is positive and she is referred for colposcopy. A LLETZ is performed and removes what is found to be a CIN 2.

When should this woman next be screened?

A. 1 month
B. 2 months
C. 3 months
D. 6 months
E. 9 months
A

D. 6 months

For a cervical intraepithelial neoplasia (essentially pre-cervical cancer) that has been removed, there should be a cervical swab 6 months later for a ‘test of cure”.

172
Q

A 37 year old woman who has not previously engaged with cervical screening presents with vaginal bleeding. A speculum exam reveals an abnormal looking cervix which is biopsied, revealing cervical cancer. An MRI is performed for staging and reveals that the cancer has spread from the cervix into the body of the uterus and the upper half of the vagina.

According to FIGO, what stage would this cancer be?

A. 2
B. 3a
C. 3b
D. 3c
E. 4
A

A. 2

Here is a simplified version of the FIGO staging of cervical cancer (each stage has several subdivisions but these are omitted):

1 - Limited to the cervix (NB: extension into the body of the uterus is disregarded)
2 - Spreads into the upper 2/3 of the vagina and the connective tissue around the uterus
3 - Spreads into the lower 1/3 of the vagina and/or involves the pelvic wall and/or causes hydronephrosis or a non-functional kidney and/or involves the para-aortic/ pelvic lymph nodes
4 - Spreads beyond the true pelvis to the mucosa of the bladder and rectum, or to distant sites

173
Q

What should be given first in post-partum haemorrhage to reduce uterine blood loss?

A

Oxytocin (Syntocinon)

174
Q

Which of these conditions would not warrant a woman taking high dose (5mg) folic acid whilst trying to conceive and throughout pregnancy?

A. BMI >30
B. Type 1 diabetes
C. Well-controlled HIV
D. Well-controlled epilepsy
E. Preeclampsia in a previous pregnancy
A

E. Preeclampsia in a previous pregnancy

Preeclampsia in a previous pregnancy is a strong risk factor for it occurring again, but folic acid supplementation has been shown to not affect this risk, therefore these women should take the standard 400mcg dose.

Diabetes of any type is linked to an increased risk of neural tube defects, though the risk is lower the better controlled it is. A BMI of over 30 is also linked to neural tube defects.

Anti-retroviral and anti-epileptic drugs are both linked with neural tube defects.

175
Q

A 28 year old woman presents with a painless, unilateral vulval swelling near to the vaginal orifice. The swelling is well demarcated and fluctuant, but does not appear inflamed.

What is the most likely diagnosis?

A. Bartholin's cyst
B. Vulval malignancy
C. Abscess
D. Benign tumour
E. Brenner's tumour
A

A. Bartholin’s cyst

This is a cyst forming in one of Bartholin’s glands - glands either side of the vaginal introitus that secrete lubrication. Blockage of the duct can cause a cyst to form. which may become infected and form an abscess. Small asymptomatic cysts may be left alone, but will require antibiotics and marsupialisation if they become infected and problematic.

176
Q

What are the investigation, non-surgical, and surgical management of stress incontinence?

A

Investigation: Assess pelvic floor muscles, perform urine dipstick, bladder diary
Non-surgical: Pelvic floor exercises
Surgical: Colposuspension or autologous rectal fascial sling

177
Q

How many weeks after an abortion should a woman take a pregnancy test to check she is not still pregnant?

A

3-4 weeks

178
Q

A 37 week pregnant g1p0 woman presents to A&E with abdominal pains that she believes is the onset of labour. The pains are fundal dominant and have been steadily increasing in frequency. Whilst being seen her membranes rupture, and an examination shows she is 4cm dilated. However the examination also shows the baby is in a transverse position.

How should this labour be managed?

A. Give nifedipine, erythromycin, and corticosteroids, and start a magnesium sulphate infusion
B. Start I.V. Syntocinon
C. Caesarean section
D. Attempt external cephalic version
E. Give nifedipine and attempt external cephalic version

A

C. Caesarean section

Her membranes have ruptured so ECV cannot be attempted. The baby cannot be vaginally delivered fro a transverse position, so a C-section is needed.

179
Q

What is the standard investigation, non-surgical, and surgical management of urge incontinence?

A

Investigation: Perform urine dipstick, bladder diary, sometimes urodynamic studies*
Non-surgical: Bladder training, anti-cholinergic medication
Surgical: Botulinim type A injection, percutaneous sacral nerve stimulation

  • can be used to diagnose urge incontinence
180
Q

What are the first and second-line treatments for dysmenorrhea?

A

1st: NSAIDs or hormonal contraception (COCP, POP, depot, implant, IUS)
2nd: A combination of the two

181
Q

What are the first and second-line treatments for menorrhagia?

A

1st: A Levonorgesterel IUS (e.g. the Mirena coil)
2nd: Oral contraception (COCP, cyclic oral progestogen), or Tranexamic acid, or an NSAID

182
Q

A 29 year old woman who is 2 weeks post-partum visits the GP with a two day history of breast pain. Her right breast is red, swollen, and tender above the nipple, though there are no palpable masses. The GP takes a set of basic obs which are normal apart from a small fever. Her baby is healthy and she has been breastfeeding him exclusively.

How should this woman be managed?

A. Tell her to stop breastfeeding, prescribe oral flucloxacillin, and take a basic set of obs for the baby
B. Tell her she needs to go and be admitted to hospital
C. Prescribe oral flucloxacillin, and advise her to stop breastfeeding until the symptoms abate
D. Make an urgent referral to a general surgeon
E. Advise her to continue breastfeeding from both breasts, use OTC analgesia, prescribe oral flucloxacillin

A

E. Advise her to continue breastfeeding from both breasts, use OTC analgesia, prescribe oral flucloxacillin

It is important in cases of mastitis for the mother to continue to breastfeed or manually express her milk, as milk stasis is a causative factor. Over the counter (OTC) analgesia is advised, and antibiotics should be given in cases where: symptoms persist for 12-24 hours, there is an infected nipple fissure, or a breast milk culture is positive.

183
Q

When does the progestogen implant (Nexplanon) have to be inserted in order to be immediately effective?

A

Within the first 5 days of a woman’s cycle

If inserted afterwards, it will take 7 days to become effective

184
Q

A woman attends hospital for her 12 week dating scan, but the sonographer cannot find an intrauterine or tubal pregnancy on ultrasound.

What is the best next step?

A. Take bloods including a serum beta-hCG, progesterone, gonadotropins, and oestriol
B. Advise the woman on what to do if she develops symptoms, and perform two beta-hCG measurements 48 hours apart
C. Counsel her that this is a missed miscarriage, and offer emotional bereavement support
D. Ask her to return in 7-14 days for another scan
E. Advise her that this is almost certainly an extrauterine pregnancy, and offer methotrextae or exploratory surgery

A

B. Advise the woman on what to do if she develops symptoms, and perform two beta-hCG measurements 48 hours apart

This is a pregnancy of unknown location (PUL), and so two sets of beta-hCG measurements 48 hours apart are taken in order to guide management. If the levels rise by >63% in that time, it is very likely there is a viable intrauterine pregnancy. These women should be offered another TVUSS in 7-14 days.

If the levels fall by >50% in that time the pregnancy is unlikely continue, and the woman should be asked to take a pregnancy test in 14 days.

If there is an intermediate result, the woman should be referred for clinical assessment within 24 hours.

NB: It is important to inform women of the symptoms of an ectopic pregnancy so they know to come to hospital if they develop.

185
Q

A 64 year old woman presents to her GP practice with a 2 week history of intermittent light vaginal bleeding. She is not sexually active, and is not taking HRT. She denies any pain and has been otherwise well. She is referred under a 2 week wait to gynaecology who peform a trans-vaginal USS which shows an endometrial thickness of 7mm.

What is the most appropriate next step?

A. Prescribe a topical oestrogen gel
B. Referral for a CTAP
C. Prescribe continuous combined HRT
D. A pelvic USS
E. A Pipelle biopsy
A

E. A Pipelle biopsy

In a post-menopausal woman presenting with vaginal bleeding, it is important to perform a trans-vaginal USS to measure the endometrial thickness. If it is above 4mm, that is suggestive of endometrial cancer, so a Pipelle (aspiration) biopsy is needed.

186
Q

Which of these is the best method of contraception for a 39 year old g1p1 woman currently 2 weeks post-partum who is exclusively breastfeeding her child?

A. Hysterectomy
B. Lactational amenorrhea method
C. Insertion of the IUS
D. Insertion of the IUD
E. COCP
A

B. Lactational amenorrhea method

It sounds surprising, but if a woman is breastfeeding her child exclusively, the prolactin produced will act as contraception. This will only work for the first 6 months, and only as long as the mother feeds the baby often and does not use formula top ups. Assuming perfect use, this method is ~98% effective.

As for the other methods:
Hysterectomy is a bit radical, especially considering she has only had one child and may want more
Insertion of either the IUS or IUD should be done within 48 hours of birth, or at 4 weeks
The COCP is absolutely contraindicated for the first 4 weeks post-partum in breastfeeding women

https://www.rcog.org.uk/globalassets/documents/guidelines/best-practice-papers/best-practice-paper-1—postpartum-family-planning.pdf

187
Q

For each of the following examples, give the gravidity and parity of the woman:

  1. A woman who is currently 18 weeks pregnant, with two previous miscarriages, and one live baby
  2. A woman who is not pregnant, but has had one stillbirth
  3. A woman who recently gave birth to a baby, and has previously had a miscarriage, and an ectopic pregnancy that was managed with salpingotomy at 7 weeks
A
  1. A woman who is currently 18 weeks pregnant, with two previous first trimester miscarriages, and one live baby - g4p1+2
  2. A woman who is not pregnant, but has had one stillbirth - g1p1
  3. A woman who recently gave birth to a baby, and has previously had a miscarriage, and an ectopic pregnancy that was managed with salpingectomy at 7 weeks - g3p1+2

Gravidity and parity can be confusing, so here is a summary:

Gravidity is the total number of pregnancies, regardless of outcome; this includes current pregnancies. Twins or multiple count as 1.

Parity is the number of pregnancies carried to the limit of viability (24 weeks). There is no consensus on whether twins are counted as 1 or 2 for parity, and the course lead (big up Roshni Patel) has said that accordingly she will not examine this debated section.

Pregnancies not carried to 24 weeks (miscarriage, ectopic, TOP) are sometimes added after the parity number (GxPx+y) though this is not always consistent
Stillbirth counts as surviving to 24 weeks and so counts for parity

This website has a good summary:
https://teachmeobgyn.com/history-taking-examinations/history-taking/obstetric/#:~:text=Gravidity%20and%20Parity,%2B0%20in%20the%20UK).

NB: This has been examined before and you should work on being confident calculating it

188
Q

Using Naegele’s rule, calculate each of these women’s due date from the dates of the first day of their last menstrual period:

  1. 1st day LMP: 9th January 2020
  2. 1st day LMP: 8th March 2020
  3. 1st day LMP 28th October 2020
A
  1. LMP: 9th January 2020 - Due date: 16th October 2020
  2. LMP: 8th March 2020 - Due date: 15th December 2020
  3. LMP 28th October 2020 - Due date: 4th August 2021
Naegele's rule is:
Take the *first day* of the last menstrual period
Add 1 year
Subtract 3 months
Add 7 days

NB: This has been examined before

189
Q

A 33 week pregnant woman presents to A&E after a mild car crash. Nobody was seriously hurt but she was thrown against the dashboard and hit it with her belly. Her obs are stable, examination of her abdomen is unremarkable, and a CTG shows that the baby is healthy.

How should this woman be managed next?

A. Take blood for a group and antibody screen, then give 500IU anti-D prophylaxis, and perform a Kleihauer test
B. Admit the woman for a minimum of 24 hours and give corticosteroids
C. Check whether the woman had anti-D prophylaxis at 28 weeks
D. Give a standard dose of 500IU anti-D prophylaxis and discharge her
E. Admit her for a minimum of 48 hours, with routine auscultation of the fetal heart rate

A

A. Take blood for a group and antibody screen, then give 500IU anti-D prophylaxis, and perform a Kleihauer test

This is a case of a potentially sensitising event after 20 weeks: in any such case it is necessary to screen for anti-D antibodies (to see if the mother has been sensitised) and give anti-D prophylaxis. A Kleihauer test - a way to quantify feto-maternal haemorrhage (FMH) - should also be performed, This quantifies maternal haemorrhage The standard dose of 500IU will be given as a minimum within 72 hours of a potentially sensitising event, but more may be required depending on the result of the Kleuhauer test.

190
Q

A woman who is 5 days post-partum attends her GP surgery after developing a vesicular rash on her trunk. During the history, she reveals she was visited by a friend a few days prior, whose child has now developed chickenpox (an infection she didn’t have as a child). She is worried about how this may affect her baby.

How should the doctor proceed?

A. Test the mother for IgM antibodies to Varicella-Zoster virus
B. Test the baby for IgG antibodies to Varicella-Zoster virus
C. Admit the baby to hospital for a course of I.V. aciclovir
D. The baby should receive prophylactic Varicella-Zoster immunoglobulin
E. No treatment is required

A

D. The baby should receive prophylactic Varicella-Zoster immunoglobulin

The risk of maternal - baby transmission of VZV is considered to be significant if the mother is infected within 4 weeks before, or 1 week after, birth. Serious infection is more likely if the mother is infected within one week either side of birth. Accordingly, this child should receive VZIG prophylaxis to prevent development of serious disease.

Testing the mother for IgM antibodies will be of limited use, the history shows fairly clearly that she is infected and the antibody test won’t add any information. Testing the baby for IgG antibodies would not be helpful because they will form after IgM antibodies to fight the infection, but the goal is to prevent infection taking hold. I.V. aciclovir would probably only be given if severe disease developed.

191
Q

According to the Abortion Act 1967, what are the 4 reasons an abortion may be carried out?

A

A) The pregnancy has not exceeded 24 weeks, and to continue would risk injury to the physical or mental wellbeing of the woman or her existing children

B) Abortion is necessary to prevent serious permanent injury to mother’s physical or mental health

C) Continuing the pregnancy would risk the mother’s life more than terminating it

D) The baby would be severely handicapped

192
Q

Under which section of the abortion act are the majority of abortions carried out?

A

A) The pregnancy has not exceeded 24 weeks, and to continue would risk injury to the physical or mental wellbeing of the woman or her existing children

97% of abortions are carried out under this sub-section

193
Q

When should labour be induced in each of: MCMA, MCDA, and DCDA pregnancies?

A

MCMA: 32 to 33+6 weeks
MCDA: 36 to 36+6 weeks
DCDA: 37 to 37+6 weeks

194
Q

Which 2 vaccines are recommended by the NHS during pregnancy?

A. Rotavirus and HPV
B. Tetanus and diptheria
C. Whooping cough and influenza
D. MMR and HiB
E. MMR and influenza
A

C. Whooping cough and influenza

195
Q

A 28 year old, 7 week pregnant woman presents with suprapubic pain. She admits to some discoloured discharge when asked, but dismissed this as an effect of pregnancy. On vaginal examination there is cervical motion tenderness and a small amount of blood in the vagina. Her observations show she is febrile at 38.4 with a RR of 15 a BP of 126/78, and a HR of 85. A gonorrhoea test is positive, and an USS shows a viable intrauterine pregnancy. Her pregnancy has so far been complicated by nausea and vomiting which started a week ago.

How should this patient be managed?

A. Employ watchful waiting for 2 weeks, then reassess
B. Give an I.M. dose of Ceftriaxone, then prescribe oral Metronidazole and Doxycycline
C. Treat her with Metronidazole as an outpatient
D. Admit her for I.V. Ceftriaxone and Erythromycin
E. Admit her for I.V. Ciprofloxacin and Doxycycline

A

D. Admit her for I.V. Ceftriaxone and Erythromycin

This is a history of pelvic inflammatory disease in a pregnant woman. PID may be caused by a range or organisms including Chlamydia trachomatis and Neisseria gonorrhoeae, and it is important to treat it as the complications can be severe (notably infertility).

There are two factors that require this woman to be admitted for treatment: she is pregnant, and she has been experiencing nausea and vomiting which may make it difficult to take oral antibiotics. Between the two choices of I.V. antibiotic regimes given here, Ciprofloxacin and Doxycycline are wrong because they are both avoided in pregnancy.

NB: Cervical excitation is a useful sign of PID, but may also indicate ectopic pregnancy, so it is important to rule out an ectopic in cases of PID

Link to PID guidelines:
https://www.bashh.org/documents/3572.pdf

196
Q

When does nausea and vomiting in pregnancy typically start and end?

A

Starts week 4-6

Ends week 16-20

197
Q

A routine cervical smear shows an area of dyskaryosis. The cervix is biopsied, which reveals a cervical intraepithelial neoplasia type 1.

How is this condition managed?

A. No treatment is needed, repeat the smear after 12 months
B. A LLETZ procedure
C. A radical hysterectomy 
D. Cervical ablation
E. Radiotherapy
A

A. No treatment is needed, repeat the smear after 12 months

CIN2 and above is generally treated with a LLETZ, but CIN1 is just monitored as it may spontaneously resolve and will not progress quickly.

198
Q

A 54 year old woman presents to her GP with vaginal bleeding. The bleeding is not associated with any pain and does not seem to be linked to sex. She reached the menopause 5 years ago and declined HRT. The GP refers her to gynaecology who perform a TVUSS which reveals an endometrial thickness of 6mm.

What is the most appropriate next step?

A. Perform a Ca-125
B. Review in 1 month
C. Refer for hysteroscopy and biopsy
D. Refer for outpatient Pipelle biopsy
E. Refer for outpatient imaging of the pelvis
A

D. Refer for outpatient Pipelle biopsy

This woman has a history that is suspicious for endometrial cancer, and her endometrial thickness is >4mm which supports this suspicion. She needs an endometrial biopsy which is done as an outpatient if possible. A hysteroscopy and biopsy is used if the woman finds the outpatient procedure intolerable, if she has cervical stenosis, or if the outpatient sample is inadequate.

Imaging (CT and MRI) would be used to stage disease if endometrial cancer was confirmed. Ca-125 is measured in primary care to assess for the presence of ovarian cancer.

199
Q

A 28 year old woman presents to A&E with a 3 day history of diarrhoea and vomiting with abdominal pain. She mentions she has also had some spotting. A vaginal exam reveals marked cervical motion tenderness.

What is the most likely diagnosis?

A. Ectopic pregnancy
B. Ovarian torsion
C. Pelvic inflammatory disease
D. Gastroenteritis
E. Urinary tract infection
A

A. Ectopic pregnancy

This is supposed to be a slightly unusual history of ectopic pregnancy because of the GI symptoms - however these are common in ectopic pregnancy and shouldn’t distract from the other clear features. The abdominal pain, spotting, and cervical motion tenderness are all features of ectopic pregnancy.

PID is also a good differential in this scenario, but the presence of bowel symptoms is consistent with ectopic pregnancy, not PID.

200
Q

What is the standard management for endometrial cancer?

A

Stage 1-2: Total abdominal hysterectomy + bilateral salpingo-oopherectomy
Stage 2+: Radical hysterectomy (means cervix is taken too)

Adjuvant chemotherapy and radiotherapy may be used for higher risk disease

201
Q

How should chronic asthma management change during pregnancy?

A

This is a bit of a trick question but important: it shouldn’t. Preventers and relievers should be taken as normal during pregnancy, especially as asthma may well worsen during pregnancy.